lời nói ñầu - lovetoan.files.wordpress.com · chứng minh rằng nếu a3 + b3 + c3 chia...

99
Nguyễn Văn Thảo Chuyên ñề Số Học - Phần I 1 Lời nói ñầu Số học là một phần rất quan trọng trong chương trình Toán phổ thông. Trong hầu hết các ñề thi học sinh giỏi thì bài Số học thường xuyên xuất hiện và luôn là một thách thức lớn ñối với học sinh. Hiện nay, không còn hệ chuyên cấp Trung học cơ sở nên các em học sinh chuyên Toán cũng không ñược học nhiều về phần này nên thường gặp rất nhiều khó khăn khi giải các bài toán ñó. Vì vậy, tôi biên soạn tài liệu này nhằm giải quyết phần nào những khó khăn ñó cho các em học sinh chuyên Toán. Chuyên ñề gồm ba chương: -Chương I. Các bài toán chia hết -Chương II. Các bài toán ñồng dư -Chương III. Các bài toán khác. Ở mỗi bài ñều ñược trình bày ba phần: Hệ thống lí thuyết; hệ thống các ví dụ và cuối cùng là hệ thống các bài tập tự giải. Các ví dụ và bài tập luôn ñược sắp xếp với ñộ khó tăng dần - theo quan ñiểm của tác giả. Tuy nhiên, do trình ñộ có hạn nên không thể tránh khỏi nhiều thiếu sót, rất mong ñược các thầy cô ñóng góp ñể hoàn thiện hơn. Xin chân thành cảm ơn! NGUYỄN VĂN THẢO

Upload: others

Post on 20-Feb-2020

3 views

Category:

Documents


0 download

TRANSCRIPT

Page 1: Lời nói ñầu - lovetoan.files.wordpress.com · Chứng minh rằng nếu a3 + b3 + c3 chia hết cho 9 thì một trong ba số a, b, c phải chia hết cho 3. Lời giải

Nguyễn Văn Thảo Chuyên ñề Số Học - Phần I

1

Lời nói ñầu Số học là một phần rất quan trọng trong chương trình Toán phổ thông. Trong hầu hết các ñề thi học sinh giỏi thì bài Số học thường xuyên xuất hiện và luôn là một thách thức lớn ñối với học sinh.

Hiện nay, không còn hệ chuyên cấp Trung học cơ sở nên các em học sinh chuyên Toán cũng không ñược học nhiều về phần này nên thường gặp rất nhiều khó khăn khi giải các bài toán ñó. Vì vậy, tôi biên soạn tài liệu này nhằm giải quyết phần nào những khó khăn ñó cho các em học sinh chuyên Toán.

Chuyên ñề gồm ba chương:

-Chương I. Các bài toán chia hết

-Chương II. Các bài toán ñồng dư

-Chương III. Các bài toán khác.

Ở mỗi bài ñều ñược trình bày ba phần: Hệ thống lí thuyết; hệ thống các ví dụ và cuối cùng là hệ thống các bài tập tự giải. Các ví dụ và bài tập luôn ñược sắp xếp với ñộ khó tăng dần - theo quan ñiểm của tác giả.

Tuy nhiên, do trình ñộ có hạn nên không thể tránh khỏi nhiều thiếu sót, rất mong ñược các thầy cô ñóng góp ñể hoàn thiện hơn. Xin chân thành cảm ơn!

NGUYỄN VĂN THẢO

Page 2: Lời nói ñầu - lovetoan.files.wordpress.com · Chứng minh rằng nếu a3 + b3 + c3 chia hết cho 9 thì một trong ba số a, b, c phải chia hết cho 3. Lời giải

Nguyễn Văn Thảo Chuyên ñề Số Học - Phần I

2

Chương I

CÁC BÀI TOÁN VỀ CHIA HẾT

I.1 Chia hết

I.1.1 Lí thuyết

I.1.1.1 ðịnh nghĩa

Cho m và n là hai số nguyên , n ≠ 0. Ta nói rằng m chia hết cho n (hay n chia hết m) nếu tồn tại một số nguyên k sao cho m = kn.

Kí hiệu: m ⋮ n, (ñọc là m chia hết cho n) hay n | m, (ñọc là n chia hết m).

I.1.1.2 Các tính chất cơ bản

Cho các số nguyên x, y, z. Ta có:

a) x ⋮ x, x ≠ 0.

b) Nếu x ⋮ y và x ≠ 0 thì |x| ≥ |y|.

c) Nếu x ⋮ z, y ⋮ z thì ax + by ⋮ z với mọi số nguyên a, b.

d) Nếu x ⋮ z và x ∓ y ⋮ z thì y ⋮ z

e) Nếu x ⋮ y và y ⋮ x thì |x| = |y|.

f) Nếu x ⋮ y và y ⋮ z thì x ⋮ z.

g) Nếu x | y và y ≠ 0 thì |y

yx

.

Chứng minh

a) x = 1.x nên x ⋮ x với mọi x ≠ 0.

b) Nếu x ⋮ y , x ≠ 0 thì tồn tại k ∈ Z sao cho x = ky, k ≠ 0

⇒ |x| = |k||y| ≥ |y| do |k| ≥ 1.

Các phần còn lại cũng khá ñơn giản, việc chứng minh xin nhường lại cho bạn ñọc.

I.1.2 Các ví dụ

Ví dụ 1. Cho n là một số tự nhiên lớn hơn 1. Chứng minh rằng

a) 2n là tổng của hai số lẻ liên tiếp.

b) 3n là tổng của ba số tự nhiên liên tiếp.

Lời giải

a) Ta có 2n = (2n-1 - 1) + (2n-1 +1) suy ra ñpcm.

b) Ta có 3n = (3n-1 - 1) + (3n-1) + (3n-1 + 1) suy ra ñpcm.

Ví dụ 2. Chứng minh rằng:

Page 3: Lời nói ñầu - lovetoan.files.wordpress.com · Chứng minh rằng nếu a3 + b3 + c3 chia hết cho 9 thì một trong ba số a, b, c phải chia hết cho 3. Lời giải

Nguyễn Văn Thảo Chuyên ñề Số Học - Phần I

3

a) nếu m – n chia hết mp + nq thì m – n cũng chia hết mq + np.

b) nếu m – n chia hết mp thì m – n cũng chia hết np.

Lời giải

Nhận xét: Hai biểu thức (mp + nq) và (mq + np) là hai biểu thức có hình thức giống như “ñối xứng loại hai” vì vậy khi xét các biểu thức loại này thường người ta kiểm tra hiệu của chúng.

a) Ta có (mp + nq) – (mq + np) = (m - n)(p - q) ⋮ (m - n)

Nên nếu (mp + nq) ⋮ (m - n) thì hiển nhiên (mq + np) ⋮ (m - n).

b) Chứng minh tương tự.

Ví dụ 3. Chứng minh rằng nếu a3 + b3 + c3 chia hết cho 9 thì một trong ba số a, b, c phải chia hết cho 3.

Lời giải

Nhận xét: Với những bài toán chứng minh a chia hết cho một số cụ thể luôn khá ñơn giản! Ta có thể xét hết các trường hợp xảy ra của số dư khi a chia cho số ñó. ( Công viêc ñó chính là xét về hệ thặng dư ñầy ñủ - ñây là tập hữu hạn nên có thể thử trực tiếp)

Giả sử không có số nào trong ba số a, b, c chia hết cho 3. Khi ñó

a = 3m ± 1; b = 3n ± 1; c = 3p ± 1

Do ñó a3 + b3 + c3 = (3m ± 1)3 + (3n ± 1)3 + (3p ± 1)3

=

9 3

9 1

9 3

9 1

A

a

a

A

+ + −

không thể chia hết cho 9.

Từ ñó suy ra ñpcm.

Ví dụ 4.

Chứng minh rằng nếu a2 + b2 chia hết cho 3 thì cả a và b ñều chia hết cho 3.

Lời giải

TH1: có 1 số không chia hết cho 3, giả sử là a

Khi ñó a = 3k ± 1; b = 3q suy ra a2 + b2 = (3k ± 1)2 + (3q)2

= 3(3k2 ± 2k + 3q2) + 1 không chia hết cho 3.

TH2: cả hai số không chia hết cho 3.

Khi ñó a = 3k ± 1; b = 3q ± 1 suy ra a2 + b2 = 3A +2

Do ñó cả a và b phải chia hết cho 3.

Ví dụ 5. Chứng minh rằng với mọi số tự nhiên chẵn n và mọi số tự nhiên lẻ k thì

Page 4: Lời nói ñầu - lovetoan.files.wordpress.com · Chứng minh rằng nếu a3 + b3 + c3 chia hết cho 9 thì một trong ba số a, b, c phải chia hết cho 3. Lời giải

Nguyễn Văn Thảo Chuyên ñề Số Học - Phần I

4

S = 1k + 2k + … + nk luôn chia hết cho n + 1.

Lời giải

Ta có 2S = (1k + nk) + (2k + (n - 1)k) + … ⋮ n + 1

Mà n chẵn nên n + 1 lẻ nên (2, n+ 1) = 1

Do ñó S ⋮ n + 1.

Ví dụ 6. Cho p là s ố nguyên tố, p > 3 v à 3

122 −=

p

n .Chứng minh rằng

n 22 ⋮−n .

Lời giải

Vì p là số nguyên tố và p>3 ⇒ )3(mod12 1 ≡−p

Mặt khác (2, p) = 1 nên theo ñịnh lí Fermat ta có

)(mod12 1 pp ≡−

Do ñó pp 312 1 ⋮−−

Ta có

n. 22 n 12n 12 3

12 n Vi

12 12 2p 1-n rasuy 3

)12)(12(4

3

441

3

121

n1-n2p2p

2p1-n

112

⋮⋮⋮

⋮⋮

−⇒−⇒−⇒−

=

−−⇒

−+=

−=−

−=−

−− pppp

n

Từ ñó suy ra ñiều phải chứng minh.

Ví dụ 7. Cho x, y là hai số nguyên khác -1 sao cho

1

1

1

1 33

++

+++

x

y

y

x là một số nguyên

Chứng minh rằng 12004 −x chia hết cho y+1.

Lời giải

Trước hết ta ñặt d

c

xb

a

y

x=

++

=++

1

1y ;

1

1 33

với a, b, c, d nguyên và b > 0, d > 0, (a,b) = 1, (c,d) = 1.

Ta có

bd

bcad

d

c

b

a +=+ nguyên

Do ñó

Page 5: Lời nói ñầu - lovetoan.files.wordpress.com · Chứng minh rằng nếu a3 + b3 + c3 chia hết cho 9 thì một trong ba số a, b, c phải chia hết cho 3. Lời giải

Nguyễn Văn Thảo Chuyên ñề Số Học - Phần I

5

bd b ad b bcad bd ⋮⋮⋮⋮ ⇒⇒+⇒+ bcad vì (a,b)=1 (1)

Mặt khác

(2) da dac bd ac

)1)(1(1

1.

1

1. 22

33

⋮⋮⋮ ⇒⇒⇒

∈+−+−=++

++

= Zyyxxx

y

y

x

d

c

b

a

Vì (c,d)=1 nên từ (1) và (2) suy ra

a⋮b suy ra b = 1 vì (a,b) = 1

(3) 1y 1 x )1(1 x 1

1 333

++⇒+=+⇒=++

⋮yab

a

y

x

Mà 1 x 1-)(x 1 366432004 +=− ⋮x

Kết hợp với (3) suy ra ñiều phải chứng minh.

Ví dụ 8. Cho n 5≥ là số tự nhiên .Chứng minh rằng

−n

n )!1( ⋮ n-1 .

Lời giải

a) Trường hợp 1. n là số nguyên tố

Theo ñịnh lý Winson (n-1)!≡ -1(mod n) suy ra ((n-1)!+1 ⋮n

Ta có

11)!1(11)!1()!1(−

+−=

−+−

=

−n

n

nn

n

n

n (vì 1

10 <<

n)

= 1)-(n )1()!1(⋮

n

nn −−− vì (n, n - 1) = 1

b) Trường hợp 2. n là hợp số

+) n không là bình phương của một số nguyên tố.

Khi ñó n = rs với 1< r < s < n.

Do (n,n-1)=1 suy ra s < n-1 ⇒ (n-1)! = kn(n-1)

suy ra

)1()1()!1(

−−=

−nnk

n

n⋮ .

+) n = p 2 với p là một số nguyên tố.

Page 6: Lời nói ñầu - lovetoan.files.wordpress.com · Chứng minh rằng nếu a3 + b3 + c3 chia hết cho 9 thì một trong ba số a, b, c phải chia hết cho 3. Lời giải

Nguyễn Văn Thảo Chuyên ñề Số Học - Phần I

6

Do p =2 n, n 5≥ suy ra p 123p 3 2 +>≥⇒≥ pp

12 2 −<⇒ pp hay 2p < n-1.

Nên 1 < p < 2p < n-1

Suy ra (n-1)! ⋮ p.2p.(n-1) = 2n(n-1).

Từ ño suy ra 1)-(n )!1(⋮

−n

n.

Vậy ta có ñiều phải chứng minh.

Ví dụ 9

Tồn tại hay không một số nguyên x sao cho 2003 12 ⋮++ xx ?

Lời giải

Ta có 2003 là số nguyên tố có dạng 3k + 2.

Giả sử tồn tại x nguyên thỏa mãn x2 + x + 1 ⋮ 2003

Từ ñó suy ra tồn tại { }2002,....,2,1∈a thỏa mãn ++ aa 2 1 ⋮ 2003 (∗ )

Ta có

2003 )1)(1(1 23 ⋮++−=− aaaa

) ( hay a a 2003mod120031 20012001 ≡−⇒ ⋮

) a ( a 2003mod2002 ≡⇒ (1)

Theo ñịnh lí Fermat ta có

2003) (mod 12002 ≡a (2)

Từ (1) và (2) ta có 2003) (mod 1≡a suy ra a = 1 (vô lí)

Vậy không tồn tại x nguyên sao thỏa mãn ñầu bài.

Ví dụ 10. (30 - 4 - 2006) Chứng minh rằng với mọi m, tồng tại một số nguyên n sao cho

n3 - 11n2 - 87n + m

Chia hết cho 191.

Lời giải

ðặt P(x) = x3 - 11x2 - 87x + m.

Ta chứng, tồn tại a, b nguyên ñể P(x) ≡ (x +a)3 + b (mod 191)

⇔ x3 + 3ax2 + 3a2x + a3 + b ≡ x3 - 11x2 - 87x + m (mod 191)

Chọn a nguyên sao cho 3a ≡ -11 (mod 191)

⇔ 3a ≡ 180 (mod 191)

⇔ a ≡ 60 (mod 191), do (3, 191) = 1,

Page 7: Lời nói ñầu - lovetoan.files.wordpress.com · Chứng minh rằng nếu a3 + b3 + c3 chia hết cho 9 thì một trong ba số a, b, c phải chia hết cho 3. Lời giải

Nguyễn Văn Thảo Chuyên ñề Số Học - Phần I

7

⇒ 3a2 ≡ 3.602 (mod 191) ≡ -87 (mod 191)

Vậy với mọi m, chỉ cần chon b ≡ m - a3 (mod 191)

là ñược P(x) ≡ (x + a)3 + b (mod 191).

Ta có, với mọi i, j nguyên thì P(i) ≡ P(j) (mod 191)

⇔ (i + a)3 ≡ (j + a)3 (mod 191)

⇒ (i + a)3.63(j + a)2 ≡ (j + a)3.63 + 2 (mod 191)

≡ (j + a) (mod 191)

⇒ (j + a)2 ≡ (i + a)189(j + a)3 (mod 191)

≡ (i + a)192 (mod 191)

≡ (i + a)2 (mod 191)

⇒ (i + a)3.63(j + a)2 ≡ (i + a)189.(i + a)2 (mod 191)

≡ i + a (mod 191)

Từ ñó suy ra

P(i) ≡ P(j) (mod 191) ⇔ i = j (mod 191)

Từ ñó suy ra tập {P(1), P(2), ..., P(191)} có 191 số dư khác nhau khi chia cho 191

Do ñó phải tồn tại một số nguyên n ∈ {1, 2, ..., n} sao cho P(n) ⋮ 191

Vậy ta có ñiều phải chứng minh.

Page 8: Lời nói ñầu - lovetoan.files.wordpress.com · Chứng minh rằng nếu a3 + b3 + c3 chia hết cho 9 thì một trong ba số a, b, c phải chia hết cho 3. Lời giải

Nguyễn Văn Thảo Chuyên ñề Số Học - Phần I

8

I.1.3 Bài tập

Bài 1. Chứng minh rằng với mọi số nguyên m, n ta có:

1) n3 + 11n ⋮ 6

2) mn(m2 – n2) ⋮ 3

3) n(n + 1)(2n + 1) ⋮ 6.

4) n3 + (n + 1)3 + (n + 2)3 ⋮ 9.

5) n2(n2 - 12) ⋮ 12

6) mn(m4 – n4) ⋮ 30

7) n5 – n ⋮ 30

8) n4 + 6n3 + 11n2 + 6n ⋮ 24

9) n4 – 4n3 – 4n2 + 16n ⋮ 384 ( n chẵn và n > 4)

10) n2 + 4n + 3 ⋮ 8

11) n3 + 3n2 – n – 3 ⋮ 48

12) n 12 – n8 – n4 + 1 ⋮ 512

13) n8 – n 6 – n4 + n2 ⋮ 1152.

14) n3 – 4n ⋮ 48 ( n chẵn)

15) n2 – 3n + 5 không chia hết cho 121.

16) (n + 1)(n + 2)…(2n) ⋮ 2n

17) n6 – n4 – n2 + 1 ⋮ 128 ( n lẻ)

Bài 2. Chứng minh rằng tích của n số nguyên lien tiếp luôn chia hết cho n!

Bài 3. Cho p là số nguyên tố lẻ. Chứng minh rằng với mọi k ∈ N, ta luôn có

S = 12k + 1 + 22k + 1 + … + (p - 1)2k + 1 chia hết cho p.

Bài 4. Chứng minh rằng nếu a3 + b3 + c3 chia hết cho 9 thì một trong ba số a, b, c phải chia hết cho 9.

Bài 5. Cho a, b nguyên. Chứng minh rằng nếu an ⋮ bn thì a ⋮ b.

Bài 6. Tìm số nguyên dương n sao cho n chia hết cho mọi số nguyên dương không vượt

quá n .

Bài 7. Chứng minh rằng a2 + b2 + c2 không thể ñồng dư với 7 modulo 8.

Bài 8. Tổng n số nguyên liên tiếp có chia hết cho n hay không? tại sao?

Bài 9. Chứng minh rằng không tồn tại cặp số nguyên (x, y) nào thỏa mãn một trong những ñẳng thức sau:

a) x2 +1 = 3y

Page 9: Lời nói ñầu - lovetoan.files.wordpress.com · Chứng minh rằng nếu a3 + b3 + c3 chia hết cho 9 thì một trong ba số a, b, c phải chia hết cho 3. Lời giải

Nguyễn Văn Thảo Chuyên ñề Số Học - Phần I

9

b) x2 + 2 = 5y.

Bài 10. Chứng minh rằng với n ≥ 1 thì

(n + 1)(n + 2) ... (n + n)

chia hết cho 2n.

Bài 11. Tìm chữ số tận cùng của số Fermat Fn = 122 +n

, n ≥ 2.

Bài 12. Tìm các số nguyên dương p, q, r sao cho

pqr - 1 ⋮ (p - 1)(q - 1)(r - 1).

Bài 13. Chứng minh rằng tồn tại một số tự nhiên có 1997 chữ số gồm toàn chữ số 1 và 2 sao cho số ñó chia hết cho 21997.

Bài 14. Cho a là một số nguyên dương và a > 2. Chứng minh rằng tồn tại vô số số nguyên dương n thỏa mãn

an - 1 ⋮ n.

Bài 15. Chứng minh rằng tồn tại vô số số nguyên dương n sao cho

2n + 1 ⋮ n.

Bài 16. Chứng minh rằng trong 12 số nguyên tố phân biệt bất kì luôn chon ra ñược 6 số a1, a2, ..., a6 sao cho

(a1 - a2)(a3 - a4)(a5 + a6) ⋮ 1800.

Bài 17. Cho a, b, c, d nguyên bất kì. Chứng minh rằng

(a - b)(a - c)(a - d)(b - c)(b - d)(c - d) ⋮ 12.

Bài 18. Tìm số tự nhiên n sao cho 2n - 1 chia hết cho 7. Chứng minh rằng với mọi số tự nhiên n thì 2n + 1 không thể chia hết cho 7.

Bài 19. Tìm số tự nhiên n sao cho n5 - n chia hết cho 120.

Bai 20. Tìm tất cả các cặp số nguyên x > 1, y > 1 sao cho

+

+

.13

13

xy

yx

Bài 21. Cho x1, x2 là hai nghiệm của phương trình x2 - mx + 1 = 0 với m là số nguyên lớn hơn 3. Chứng minh rằng với mọi số nguyên dương n thì Sn =

nn xx 21 + là một số nguyên và không chia hết cho m - 1.

Bài 22. Tìm tất cả các cặp số nguyên dương a, b sao cho

2

22

+−

ab

a

là một số nguyên.

Bài 23. (30.4.2003) Tìm ba số nguyên dương ñôi một phân biệt sao cho tích của hai số bất kì ñều chia hết cho số thứ 3.

Page 10: Lời nói ñầu - lovetoan.files.wordpress.com · Chứng minh rằng nếu a3 + b3 + c3 chia hết cho 9 thì một trong ba số a, b, c phải chia hết cho 3. Lời giải

Nguyễn Văn Thảo Chuyên ñề Số Học - Phần I

10

B ài 24. Chứng minh rằng với mọi số tự nhiên n thì giữa n2 và (n + 1)2 luôn tồn tại ba số tự nhiên phân biệt a, b, c sao cho a2 + b2 ⋮ c2.

Bài 25. Cho số tự nhiên An = 19981998...1998 (gồm n số 1998 viết liền nhau)

a) Chứng minh rằng tồn tại số nguyên dương n < 1998 sao cho An ⋮ 1999.

b) Gọi k là số nguyên dương nhỏ nhất sao cho Ak ⋮ 1999. Chứng minh rằng

1998 ⋮ 2k.

Bài 26. Cho hai số nguyên dương m và n sao cho n + 2 ⋮ m. Hãy tính số các bộ ba số nguyên dương (x, y, z) sao cho x + y + z ⋮ m trong ñó mỗi số x, y, z ñều không lớn hơn n.

Bài 27. (APMO 98) Tìm số nguyên dương n lớn nhất sao cho n chia hết cho mọi số

nguyên dương nhỏ hơn 3 n .

Bài 28. Tìm tất cả các số nguyên dương m, n sao cho n3 + 1 chia hết cho mn - 1.

Bài 29. Tìm tất cả các cạp số nguyên dương a, b sao cho

12

2

−+

ab

ba

là một số nguyên.

Bài 30. Tìm tất cả các cặp số nguyên dương sao cho

72

2

++++

bab

baba

là một số nguyên.

Bài 31. Cho n là số nguyên dương lớn hơn 1. p là một ước nguyên tố của số Fermat Fn. Chứng minh rằng p - 1 chia hết cho 2n+2.

Bài 32. Cho x, y , p là các số nguyên và p > 1 sao cho x2002 và y2002 ñều chia hết cho p. Chứng minh rằng 1 + x + y không chia hết cho p.

Bài 33. (USA - 98) Chứng minh rằng với mỗi số nguyên dương n ≥ 2, tồn tại một tập hợp n số nguyên sao cho với hai số a, b bất kì (a ≠ b) thuộc tập ñó thì (a - b)2 chia hết ab.

Bài 34. Giả sử tập S = {1, 2, 3, ..., 1998} ñược phân thành các cặp rời nhau

{ai, bi| 1 ≤ i ≤ 1998} sao cho |ai - bi| bằng 1 hoặc bằng 6. Chứng minh rằng

∑=

−999

1

||i

ii ba = 10k + 9.

Bài 35. Tìm tất cả các cặp số nguyên dương a, b sao cho

2

22

+−

ab

a

là một số nguyên.

Bài 36. Chứng minh rằng với mọi n ∈ N* luôn tồn tại số tự nhiên a sao cho

Page 11: Lời nói ñầu - lovetoan.files.wordpress.com · Chứng minh rằng nếu a3 + b3 + c3 chia hết cho 9 thì một trong ba số a, b, c phải chia hết cho 3. Lời giải

Nguyễn Văn Thảo Chuyên ñề Số Học - Phần I

11

64a2 + 21a + 7 ⋮ 2n.

Bài 37. (Nga - 1999) Cho tập A là tập con của tập các số tự nhiên n sao cho trong 1999 số tự nhiên liên tiếp bất kì luôn có ít nhất một số thuộc A.

Chứng minh rằng tồn tại hai số m, n thuộc A sao cho m ⋮ n.

Bài 38. Tìm x, y, z nguyên dương và x < y < z sao cho

2x + 2y + 2z = 2336.

Bài 39. Cho x, y, z là các số nguyên dương thỏa mãn

(x - y)(y - z)(z - x) = x + y + z.

Chứng minh rằng x + y + z chia hết cho 27.

Bài 40. Cho m, n là các số nguyên dương sao cho 4

2nm ≤ và mọi ước số nguyên tố của m

ñều nhỏ hơn hoặc bằng n. Chứng minh rằng

n! ⋮ m.

Bài 41. Tìm tất cả các cặp số nguyên dương a, b sao cho

ba

ab

ab

ba

−+

−+

2

2

2

2

;

là các số nguyên.

Bài 42. Cho x, y là hai số nguyên dương sao cho x2 + y2 + 1 chia hết cho xy.

Chứng minh rằng

.3122

=++

xy

yx

Bài 43. Cho hàm số f(x) = x3 + 17. Chứng minh rằng mỗi số nguyên dương n luôn tồn tại một số nguyên dương x sao cho f(x) chia hết cho 3n nhưng không chia hết cho 3n + 1.

Bài 44. Cho p là số nguyên tố lớn hơn 2 và p - 2 chia hết cho 3. Chứng minh rằng trong tập hợp các số có dạng x3 - y2 + 1, với x, y là các số nguyên không âm nhỏ hơn p có nhiều nhất p - 1 số chia hết cho p.

Bài 45. Chứng minh rằng với mọi n nguyên dương luôn tồn tại một số tự nhiên có n chữ số chia hết cho 2n và số này chỉ gồm các chữ số 1 và 2.

Bài 46. Cho số nguyên dương n > 1, thỏa mãn 3n - 1 chia hết cho n. Chứng minh rằng n là số chẵn.

Page 12: Lời nói ñầu - lovetoan.files.wordpress.com · Chứng minh rằng nếu a3 + b3 + c3 chia hết cho 9 thì một trong ba số a, b, c phải chia hết cho 3. Lời giải

Nguyễn Văn Thảo Chuyên ñề Số Học - Phần I

12

I.2 Ước số chung lớn nhất - Bội số chung nhỏ nhất

I.2.1. Lí thuyết

I.2.1.1. Ước số chung lớn nhất

I.2.1.1.1 ðịnh nghĩa 1

Cho a, b là hai số nguyên. Số nguyên dương d lớn nhất chia hết cả a và b ñược gọi là ước chung lớn nhất của a và b.

Kí hiệu: d = (a, b) hoặc d = gcd(a, b)

Nếu d = 1 thì ta nói a và b là hai số nguyên tố cùng nhau.

I.2.1.1.2. Các tính chất của ước chung lớn nhất

a) Nếu p là một số nguyên tố thì (p,m) = p hoặc (p, m) = 1.

b) Nếu (a, b) = d thì a = dm, b = dn và (m, n) = 1.

c) Nếu (a, b) = d, a = d’m, b = d’n và (m, n) = 1 thì d = d’.

d) Nếu m là một ước chung của a và b thì m | (a,b).

e) Nếu px || m và py || n thì pmin(x,y) || (m, n).

f) Nếu a = bq + r thì (a, b) = (r, b).

g) Nếu c | ab và (a,c) = 1 thì c | b.

h) Nếu (a, c) = 1 thì (ab, c) = (b, c).

I.2.1.2 Bội số chung nhỏ nhất.

I.2.1.2.1 ðịnh nghĩa.

Cho a, b là hai số nguyên. Số nguyên dương nhỏ nhất chia hết cho cả a và b ñược gọi là bội số chung nhỏ nhất của a và b.

Kí hiệu: [a,b] hay lcm(a,b).

I.2.1.2.2. Các tính chất của bội chung nhỏ nhất.

a) Nếu [a, b] = m và m = a.a’ = b.b’ thì (a’, b’) = 1.

b) Nếu m’ = a.a’ = b.b’ và (a’, b’) = 1 thì [a, b] = m.

c) Nếu m = [a, b] và m’ là một bội chung của a và b thì m | m’.

d) Nếu a | m và b | m thì [a, b] | m.

e) Cho n là một số nguyên dương, ta luôn có n[a, b] = [na, nb].

f) Nếu a = 1 2 1 21 2 1 2. ... ; . ...k kn mn n m m

k kp p p b p p p=

Page 13: Lời nói ñầu - lovetoan.files.wordpress.com · Chứng minh rằng nếu a3 + b3 + c3 chia hết cho 9 thì một trong ba số a, b, c phải chia hết cho 3. Lời giải

Nguyễn Văn Thảo Chuyên ñề Số Học - Phần I

13

Thì [a, b] = min( , )

1

i i

kn m

ii

p=∏ .

I.2.1.3 ðịnh lí Bézout

Phương trình mx + ny = (m,n) luôn có vô số nghiệm nguyên.

Nhận xét: Phương trình ax + by = c có nghiệm nguyên khi và chỉ khi c là

bội của (a, b).

Phương trình ax + by = 1 có nghiệm nguyên khi và chỉ khi

(a, b) = 1.

I.2.1.4 Mối quan hệ giữa ước số chung lớn nhất và bội số chung

nhỏ nhất

Cho a và b là các số nguyên khác 0, ta có

[a,b]( , )

ab

a b=

I.2.2 Các ví dụ

Ví dụ 1. Chứng minh rằng với mọi số nguyên a, b ta luôn có

(3a + 5b, 8a + 13b) = (a, b).

Lời giải

Ta có (3a + 5b, 8a + 13b) = (3a + 5b, 8a + 13b – 2(3a + 5b))

= (3a + 5b, 2a + 3b) = (a + 2b, 2a + 3b)

= (a + 2b, b) = (a, b).

ðpcm.

Ví dụ 2. Nếu (a, b) = d thì (a +b, a - b) có thể nhận nhũng giá trị nào?

Lời giải

Ta có m = (a + b, a - b) = (a + b, 2a) = (a + b, 2b).

Do ñó m là ước chung của 2a và 2b và a + b.

Nếu a + b lẻ thì (a + b, a - b) = d

Nếu a + b chẵn thì (a + b, a - b) = 2d.

Ví dụ 3. Chứng minh rằng phân số sau tối giản

21 4

14 3

n

n

++

Lời giải

Ta có (21n + 4, 14n + 3) = (7n + 1, 14n + 3)

Page 14: Lời nói ñầu - lovetoan.files.wordpress.com · Chứng minh rằng nếu a3 + b3 + c3 chia hết cho 9 thì một trong ba số a, b, c phải chia hết cho 3. Lời giải

Nguyễn Văn Thảo Chuyên ñề Số Học - Phần I

14

= (7n + 1, 14n + 3 – 2(7n + 1))

= (7n +1, 1) = 1

Từ ñó suy ra ñiều phải chứng minh.

Ví dụ 4. Cho a, b là các số nguyên dương phân biệt sao cho ab(a + b) chia hết cho a2 + ab + b2. Chứng minh rằng

3| |a b ab− >

Lời gải

ðặt g = (a, b) ⇒ a = xg và b = yg với (x, y) = 1.

Khi ñó

2 2 2 2

( ) ( )ab a b gxy x y

a ab b x xy y

+ +=

+ + + +

là một số nguyên.

Ta có (x2 + xy + y2, x) = (y2, x) = 1

(x2 + xy + y2, y) = 1.

Vì (x + y, y) = 1 nên ta có

(x2 + xy + y2, x + y) = (y2, x + y) = 1

Do ñó x2 + xy + y2 | g

Suy ra g ≥ x2 + xy + y2

Mặt khác |a - b|3 = g3|x - y|3

= g2|x - y|3g

≥ g2.1. (x2 + xy + y2) = ab.

Từ ñó ta có ñiều phải chứng minh.

Ví dụ 5. Cho n là một số nguyên dương, d = (2n + 3, n + 7).

Tìm giá trị lớn nhất của d.

Lời giải

Ta có

(2n + 3, n + 7) = (2(n + 7) – 2n -3, n + 7)

= (11, n + 7) ≤ 11.

Mặt khác khi n = 11k + 4 thì n + 7 = 11(k + 1)

⇒ (11, n + 4) = 11.

Do ñó giá trị lớn nhất của d là 11.

Page 15: Lời nói ñầu - lovetoan.files.wordpress.com · Chứng minh rằng nếu a3 + b3 + c3 chia hết cho 9 thì một trong ba số a, b, c phải chia hết cho 3. Lời giải

Nguyễn Văn Thảo Chuyên ñề Số Học - Phần I

15

Ví dụ 6. (India 1998) Tìm tất cả các bộ (x, y, n) nguyên dương sao cho

(x, n+1) = 1 (1)

và xn + 1 = yn+1. (2)

Lời giải

Từ (2) ta có

xn = yn+1 – 1

= (y - 1)(yn + yn-1 + …+ 1) (*)

ðặt m = yn + yn-1 + … + 1

Suy ra xn ⋮ m

Mà (x, n+1) = 1 nên ta phải có (m, n +1) = 1

Ta lại có

m = yn – yn-1 + 2(yn-1 – yn-2) + …+ n(y - 1) +n + 1

= (y – 1)(yn-1 + 2yn-2 + … + n) + n + 1

⇒ n + 1 ⋮ (m, y - 1)

Mà (m, n + 1) = 1 ⇒ (m, y - 1) = 1 (**)

Từ (*) và (**) suy ra m phải là luỹ thừa n của một số nguyên dương.

Tức là m = qn với q là một số nguyên dương nào ñó

Vì y > 0 nên ta có yn < yn + yn-1 + … + 1 < yn + 1 1 ...n nn nC y C− + + với mọi n > 1

hay yn < qn < (y + 1)n với mọi n > 1 (vô lí)

Vậy n = 1 ⇒ x = y2 – 1

Vì (x, n +1) = (x, 2) = 1 nên x = 2k + 1 ⇒ y chẵn

Do ñó (x, y, n ) = (4a2- 1, 2a, 1) với a nguyên dương.

Ví dụ 7. Chứng minh rằng nếu một số nguyên dương có số ước số là lẻ thì ñó phải là số chính phương.

Lời giải

Gọi n là số tự nhiên như vậy.

Nhận thấy, nếu d là một ước số của n thì n

d cũng là một ước số của n.

Do vậy nếu với mọi d mà d ≠n

d thì số ước của n phải là chẵn.

Nên tồn tại d là ước của n sao cho d = n

d ⇔ n = d2 (ñpcm).

Page 16: Lời nói ñầu - lovetoan.files.wordpress.com · Chứng minh rằng nếu a3 + b3 + c3 chia hết cho 9 thì một trong ba số a, b, c phải chia hết cho 3. Lời giải

Nguyễn Văn Thảo Chuyên ñề Số Học - Phần I

16

Ví dụ 8. (APMO - 1999) Tìm số nguyên dương n lớn nhất sao cho n chia hết cho mọi số

tự nhiên nhỏ hơn 3 n .

Lời giải

Câu trả lời là 420.

Thật vậy, ta có

[1, 2, 3, 4, 5, 6, 7] = 420. 7 < .84203 <

Giả sử n > 420 và thỏa mãn ñiều kiện ñầu bài ⇒ 73 >n ⇒ n ⋮ 420.

Do ñó n ≥ 2.420 = 480 ⇒ 93 ≥n .

Ta có

[1, 2, ..., 9] = 2520 ⇒ n ⋮ 2520 ⇒ 133 >n .

Gọi m là số nguyên dương lớn nhất nhỏ hơn 3 n ⇒ n ≥ 13 và m3 < n ≤ (m +1)3.

Do n ⋮ [1, 2, ..., m] ⇒ n ⋮ [m - 3, m - 2, m - 1, m]

Mặt khác

6

)3)(2)(1(],1,2,3[

−−−≥−−−

mmmmmmmm

nên

3)1(6

)3)(2)(1(+≤≤

−−−mn

mmmm

suy ra

)3)(2)(1(

)1(6 3

−−−+

≤mmm

mm

⇔ )3

41)(

2

31)(

1

21(6

−+

−+

−+≤

mmmm

⇔ )3

41)(

2

31)(

1

21(6)(

−+

−+

−+−=

mmmmmf ≤ 0

Mở rộng tập xác ñịnh của m trên tập số thực ta dễ dàng chứng minh ñược f(m) là hàm số ñồng biến trên tập [13; +∞)

Do ñó với mọi m ≥ 13 thì f(m) ≥ f(13) > 0 (vô li)

nên ñiều giả sử là sai.Từ ñó suy ra ñiều phải chứng minh.

Page 17: Lời nói ñầu - lovetoan.files.wordpress.com · Chứng minh rằng nếu a3 + b3 + c3 chia hết cho 9 thì một trong ba số a, b, c phải chia hết cho 3. Lời giải

Nguyễn Văn Thảo Chuyên ñề Số Học - Phần I

17

I.2.3 Bài tập

Bài 1. Cho m, n là hai số nguyên dương phân biệt và (m, n) = d.

Tính (2006m + 1, 2006n + 1).

Bài 2. Chứng minh rằng nếu các số a, b, c ñôi một nguyên tố cùng nhau thì

(ab + bc + ca, abc) = 1.

Bài 3. Tìm

a) (21n + 4, 14n + 3)

b) (m3 + 2m, m4 + 3m2 + 1)

c) [2n - 1, 2n + 1].

Bài 4. Chứng minh rằng (2p - 1, 2q - 1) = 2(p, q) - 1.

Bài 5. Cho a, m, n là các số nguyên dương, a > 1 và (m, n) = 1. Chứng minh rằng

(a - 1)(amn - 1) ⋮ (am - 1)(an - 1).

Bài 6. Chứng minh rằng

[1, 2, ..., 2n] = (n +1, n + 2, ..., 2n)

Bài 6. Chứng minh rằng với mọi số nguyên dương m >n ta có

[ ] [ ]nm

mnnmnm

−>+++

21,1, .

Bài 7. Chứng minh rằng dãy 1, 11, 111, ... chứa vô hạn cặp (xn, xm) nguyên tố cùng nhau.

Bài 8. Cho n là một số nguyên dương, a và b nguyên dương và nguyên tố cùng nhau.

Chứng minh rằng ),( baba

ba nn

−−−

bằng 1 hoặc n.

Bài 9. Cho m, n là các số nguyên dương, a là một số nguyên dương lớn hơn 1.

Chứng minh rằng

.1)1,1( ),( −=−− nmnm aaa

Bài 10. (Hàn Quốc 1998) Tìm tất cả các số nguyên dương l, m, n ñôi một nguyên tố cùng nhau sao cho

)111

)((nml

nml ++++

là một số nguyên.

Bài 11. (Canada - 97)Tìm số các cặp số nguyên a, b (a ≤ b) thoả mãn

Page 18: Lời nói ñầu - lovetoan.files.wordpress.com · Chứng minh rằng nếu a3 + b3 + c3 chia hết cho 9 thì một trong ba số a, b, c phải chia hết cho 3. Lời giải

Nguyễn Văn Thảo Chuyên ñề Số Học - Phần I

18

[ ]

=

=

!50,

!5),(

ba

ba

Bài 12. (Hungari - 1998) Tìm n nguyên dương sao cho tồn tại các cặp số nguyên a, b thỏa mãn

(a, b) = 1998 và [a, b] = n!

Bài 13. (Nga - 2000) Cho 100 số nguyên dương nguyên tố cùng nhau xếp trên một vòng tròn. Xét phép biến ñổi như sau: Với mỗi số nguyên trên vòng tròn ta có thể cộng thêm ước chung lớn nhất của hai số kề bên nó. Chứng minh rằng sau một số hữu hạn phép biến ñổi ñó, ta có thể thu ñược các số mới ñôi một nguyên tố cùng nhau.

Bài 14. (Hungari - 1997) Cho tập A gồm 1997 số nguyên phân biệt sao cho bất kì 10 số nào trong A cũng có bội chung nhỏ nhất như nhau. Tìm số lớn nhất các số ñôi một nguyên tố cùng nhau có thể có trong A.

Bài 15. Cho s và t là các số nguyên dương khác 0. Với cặp (x, y) bất kì, gọi T là phép biến ñổi (x, y) thành cặp (x - t, y - s). Cặp (x, y) ñược gọi là “Tốt” nếu sau hữu hạn phép biến ñổi T ta thu ñược cặp mới nguyên tố cùng nhau.

a) Tìm (s, t) sao cho (s, t) là một cặp “Tôt”

b) Chứng minh rằng với mọi s, t thì luôn tồn tại cặp (x, y) không “Tôt”.

Bài 16. Tồn tại hay không các cặp số nguyên dương a, b sao cho

(30a + b)(30b + a) = 42001?

Page 19: Lời nói ñầu - lovetoan.files.wordpress.com · Chứng minh rằng nếu a3 + b3 + c3 chia hết cho 9 thì một trong ba số a, b, c phải chia hết cho 3. Lời giải

Nguyễn Văn Thảo Chuyên ñề Số Học - Phần I

19

I.3. Số nguyên Tố

I.3.1. Lí thuyết

I.3.1.1 ðịnh nghĩa Một số nguyên dương p ñược gọi là số nguyên tố, nếu nó chỉ có hai ước số dương là 1 và chính nó.

Nếu p không phải số nguyên tố thì p ñược gọi là hợp số.

Nhận xét: 2 là số nguyên tố chẵn duy nhất.

I.3.1.2 ðịnh lý 1 ( ðịnh lý cơ bản của số học)

Mọi số tự nhiên lớn hơn 1 ñều có thể phân tích một cách duy nhất thành tích các thừa số nguyên tố.

I.3.1.3. ðịnh lý 2

Tập hợp các số nguyên tố là vô hạn.

I.3.1.3. ðịnh lý 3

Cho p là một số nguyên tố. Nếu p | ab thì p | a hoặc p | b.

(Việc chứng minh các ñịnh lý trên khá ñơn giản và ta có thể tìm ñược trong bất kì một quyển sách số học nào, vì vậy sẽ không ñược trình bày tại ñây)

Page 20: Lời nói ñầu - lovetoan.files.wordpress.com · Chứng minh rằng nếu a3 + b3 + c3 chia hết cho 9 thì một trong ba số a, b, c phải chia hết cho 3. Lời giải

Nguyễn Văn Thảo Chuyên ñề Số Học - Phần I

20

I.3.2 Các ví dụ

Ví dụ 1. Tìm tất cả các số nguyên dương n sao cho các số 3n – 4, 4n – 5, 5n – 3 ñều là các số nguyên tố.

Lời giải

Ta có (3n - 4) + (5n - 3) = 8n – 7 là số lẻ

Do ñó trong hai số trên phải có một số chẵn và một số lẻ.

Nếu 3n – 4 chẵn thì 3n – 4 = 2 ⇔ n = 2 ⇒ 4n – 5 = 3 và 5n – 3 = 7 ñều là các số nguyên tố.

Nếu 5n – 4 chẵn thì 5n – 3 = 2 ⇔ n = 1 ⇒ 3n – 4 = -1 (loại)

Vậy n = 2.

Ví dụ 2. Tìm số nguyên tố p sao cho 8p2 + 1 và 8p2 – 1 cũng là những số nguyên tố.

Lời giải

Nếu p = 2 thì 8p2 + 1 = 33 ⋮ 3 nên không thỏa mãn.

Nếu p = 3 thì 8p2 + 1 = 73 và 8p2 – 1 = 71 ñều là số nguyên tố nên p = 3 thỏa mãn

Nếu p > 3 và p nguyên tố nên p không chia hết cho 3.

Do ñó p = 3k + 1 hoặc p = 3k – 1

+) p = 3k + 1 ⇒ 8p2 + 1 = 8(3k + 1)2 + 1

= 72k2 + 48k + 9 ⋮ 3

Và hiển nhiên 8p2 + 1 > 3 nên 8p2 + 1 là hợp số.

+) p = 3k – 1 ⇒ 8p2 + 1 ⋮ 3 và 8p2 – 1 > 3 nên không thỏa mãn.

Vậy p = 3.

Ví dụ 3. Cho p ≥ 5 thỏa mãn p và 2p + 1là số nguyên tố. Chứng minh rằng 4p + 1 là hợp số.

Lời giải.

+) p = 3k + 2 ⇒ 4p + 1 = 4(3k + 2) + 1 = 12k + 9 ⋮ 53⇒ 4p + 1 là hợp số.

+ p = 3k + 1 ⇒ 2p + 1 = 2(3k + 1) + 1 = 6k + 3 ⋮ 3 (vô lí vì 2p + 1 là số nguyên tố lớn hơn 11.)

Ví dụ 4. Tìm số nguyên tố p sao cho 2p + 1 là lập phương của một số tự nhiên.

Lời giải

Ta có 2p + 1 = n3 ⇔ 2p = n3 – 1 = (n - 1)(n2 + n + 1) (*)

Do với mọi số tự nhiên n thì n2 + n + 1 > n – 1 và mọi số nguyên tố p thì p ≥ 2

Page 21: Lời nói ñầu - lovetoan.files.wordpress.com · Chứng minh rằng nếu a3 + b3 + c3 chia hết cho 9 thì một trong ba số a, b, c phải chia hết cho 3. Lời giải

Nguyễn Văn Thảo Chuyên ñề Số Học - Phần I

21

Nên từ (*) ta có 2

1 2

1

n

n n p

− =

+ + =

Từ ñó tìm ñược p = 13.

Ví dụ 5. Chứng minh rằng với mọi số nguyên dương a > 2, tồn tại vô số số nguyên dương n sao cho: an – 1 ⋮ n.

Lời giải

Xét dãy số x0 = 1, xn+1 = 1nxa −

Ta sẽ chứng minh với mọi k ∈ N thì 1kxa k− ⋮ (*)

+) n = 0, hiển nhiên (*)ñúng.

+) Giả sử (*)ñúng tới n ≤ k, ta có

1 ( 1)

1

1 . 1 1 1

( ) 1 1

xkk x k

k k

x x mxak

x xmk

a m x a a a

a a x

+ −

+

− = ⇒ − = − = −

= − − =⋮

Từ ñó suy ra (*)ñược chứng minh.

Do a > 2 nên (xn) là dãy số tăng suy ra (xn) là dãy số vô hạn.

ðpcm.

Ví dụ 6. Cho p, q là hai số nguyên tố phân biệt. Chứng minh rằng 1 1 1q pp q pq− −+ − ⋮ .

Lời giải

Do p và q là hai số nguyên tố phân biệt nên (p, q) = 1

Theo ñịnh lí Fermat nhỏ ta có pq – 1 – 1 ⋮ q ⇒ pq-1 + qp-1 – 1 ⋮ q.

Tương tự ta cũng có pq-1 + qp-1 – 1 ⋮ q

Do ñó ta có ñiều phải chứng minh.

Ví dụ 7. Biết rằng 2n – 1 là một số nguyên tố. Chứng minh rằng n là số nguyên tố.

Lời giải

Do 2n – 1 là số nguyên tố nên n > 1.

Giải sử n là hợp số

Khi ñó n = pq trong ñó p và q ñều lớn hơn 1 ⇒ 2n – 1 = 2pq – 1 ⋮ 2p – 1 và 2q – 1

Mà 2p – 1 và 2q – 1 ñều lớn hơn 1 nên 2n – 1 không phải số nguyên tố (mâu thuẫn)

Vậy ta có ñiều phải chứng minh.

Ví dụ 8. Chứng minh rằng an + 1 (a và n nguyên dương) là số nguyên tố thì n =2k.

Lời giải

Page 22: Lời nói ñầu - lovetoan.files.wordpress.com · Chứng minh rằng nếu a3 + b3 + c3 chia hết cho 9 thì một trong ba số a, b, c phải chia hết cho 3. Lời giải

Nguyễn Văn Thảo Chuyên ñề Số Học - Phần I

22

Giả sử n ≠ 2k thì n = 2m.q trong ñó q là một số nguyên dương lẻ .

Khi ñó an + 1 = 2 2 2 2 ( 1)1 ( ) 1 ( 1)( ... 1)m m m mq q qa a a a −+ = + = + + +

Suy ra an + 1 không thể là số nguyên tố (Mâu thuẫn)

Vậy ta có ñiều phải chứng minh.

Ví dụ 9. Tìm tất cả các số nguyên tố p và q sao cho p + q = (p - q)3.

Lời giải

Vì (p - q)3 = p + q ≠ 0 nên p và q phân biệt và (p, q) = 1.

Mặt khác ta lại có p – q ≡ 2p (mod p + q)

Suy ra 8p3 chia hết cho p + q

Lại có 1 = (p, q) = (p + q, p) do ñó p3 và p + q cũng nguyên tố cùng nhau

Nên 8 ⋮ p+q ⇒

2

4

8

, 8.

p q

p q

p q

p q

+ = + = + = <

Thử trực tiêp ñược p = 5, q = 3.

Ví dụ 10. (Baltic 2001). Cho a là số nguyên dương lẻ, m và n là hai số nguyên dương

phân biệt. Chứng minh rằng: 2 2 2 2( 2 , 2 ) 1n n m m

a a+ + = .

Lời giải

Giả sử m > n > 0.

Gọi p là một ước nguyên tố của 2 2 2 22 2 (mod )n n n n

a a p+ ⇒ ≡ − (*)

Bình phương hai vế của (*) m – n lần ta ñược:

2 22 (mod )m m

a p≡

Do ñó 2 2 22 2.2 (mod )m m m

a p+ ≡ nên p không thể là ước của 2 22m m

a + .

Từ ñó suy ra ñiều phải chứng minh.

Ví dụ 11. (Nga - 2001) Tìm số nguyên dương lẻ n > 1 sao cho a và b là hai ước nguyên tố cùng nhau bất kì của n thì a + b – 1 cũng là ước của n.

Lời giải

TH1: n = pk trong ñó p là một số nguyên tố thì n thỏa mãn yêu cầu.

TH2: n không là luỹ thừa của một số nguyên tố.

Gọi p là một ước nguyên tố nhỏ nhất của n. Khi ñó n = pk.s và (p,s) = 1.

⇒ p + s – 1 | n

Page 23: Lời nói ñầu - lovetoan.files.wordpress.com · Chứng minh rằng nếu a3 + b3 + c3 chia hết cho 9 thì một trong ba số a, b, c phải chia hết cho 3. Lời giải

Nguyễn Văn Thảo Chuyên ñề Số Học - Phần I

23

Gọi q là một ước nguyên tố của s thì q > p.

Dễ thấy s < s + p – 1 < s + q ⇒ s + p – 1 không thể chia hết cho q

Từ ñó suy ra (s, s + p - 1) = 1. Do ñó s + p – 1 chỉ có ước nguyên tố là p

⇒ s + p – 1 = pc ⇒ s = pc – p + 1

Do pc | n và (pc, s) = 1 nên pc + s – 1 là ước của n

⇒ 2pc – p | n

Vì (2.pc – 1, p) = 1 nên 2pc – 1 là ước của s.

Mặt khác ta dễ dàng chứng minh ñược

1 1 1

2 2 1 2

c

c

p p p p

p

− − + +< <

Hay

1 1

2 2 1 2c

p s p

p

− +< <

− =

11

2

p −+

Suy ra s không thể chia hết cho 2pc – 1.

(mâu thuẫn với giả thiết)

Vậy n = pk với p là số nguyên tố.

Page 24: Lời nói ñầu - lovetoan.files.wordpress.com · Chứng minh rằng nếu a3 + b3 + c3 chia hết cho 9 thì một trong ba số a, b, c phải chia hết cho 3. Lời giải

Nguyễn Văn Thảo Chuyên ñề Số Học - Phần I

24

I.3.3 Bài tập

Bài 1. Cho a ∈ N*. Chứng minh rằng nếu am + 1 là số nguyên tố thì m = 2n. ðiều ngược lại có ñúng không?

Bài 2. Giả sử phương trình x2 + ax + b + 1 = 0 (a, b ∈ Z) có nghiệm nguyên. Chứng minh rằng a2 + b2 là hợp số.

Bài 3. Cho a, b, c là các số nguyên khác 0 và a ≠ c thỏa mãn

22

22

bc

ba

c

a

++

=

Chứng minh rằng a2 + b2 + c2 không phải là số nguyên tố.

Bài 4. Tìm n sao cho n4 + 4n là một số nguyên tố.

Bài 5. Cho p là số nguyên tố. Chứng minh rằng số

� ������ppp

9...99...2...221...11 - 123456789

chia hết cho p.

Bài 6. Tìm số tự nhiên n sao cho

A = n2005 + n2006 + n2 + n + 2

là một số nguyên tố.

Bài 7. Tìm n nguyên dương ñể mỗi số sau ñây là số nguyên tố:

a) n4 + 4

b) n4 + n2 +1.

Bài 8. Chứng minh rằng nếu p là một số nguyên tố lớn hơn 3 thì

p2 - 1 ⋮ 24.

Bài 9. Cho 2m - 1 là một số nguyên tố. Chứng minh rằng m là một số nguyên tố.

Bài 10. Tìm số nguyên tố p sao cho 2p + 1 = a3, với a nguyên dương.

Bài 11. Tìm số nguyên tố p sao cho p + 4 và p + 8 cũng là số nguyên tố.

Bài 12. Tìm số nguyên tố p sao cho 8p2 + 1 và 8p2 - 1 là những số nguyên tố.

Bài 13. Cho p là một số nguyên tố và p = 30k + r. Chứng minh rằng

r = 1 hoặc r là một số nguyên tố.

Bài 14. Cho a ∈ N*, a > 1. Chứng minh rằng an + 1 là một số nguyên tố thì n = 2k.

Bài 15. (Iran 1998) Cho a, b, x là các số nguyên dương thỏam mãn

xa + b = abb.

Chứng minh rằng x = a và b = xx.

Page 25: Lời nói ñầu - lovetoan.files.wordpress.com · Chứng minh rằng nếu a3 + b3 + c3 chia hết cho 9 thì một trong ba số a, b, c phải chia hết cho 3. Lời giải

Nguyễn Văn Thảo Chuyên ñề Số Học - Phần I

25

Bài 16. Chứng minh rằng tồn tại một dãy vô hạn {pn} các số nguyên tố phân biệt sao cho pn ≡ 1 (mod 1999n) với mọi n = 1, 2, ...

Bài 17. Tìm tất cả các số nguyên tố p sao cho

f (p) = (2 + 3) - (22 + 32) + ... - (2p-1 + 3p - 1) + (2p + 3p) chia hết cho 5.

Bài 18. (Trung Quốc 2001) Cho các số nguyên dương a, b, c sao cho a, b, c, a + b - c, c + a - b, b + c - a và a + b + c là bảy số nguyên tố phân biệt. d là số các số nguyên phân biệt nằm giữa số bé nhất và lớn nhất trong bảy số ñó. Giả sử rằng số 800 là một phần tử của tập {a + b, b + c, c + a}. Tìm giá trị lớn nhất của d.

Bài 19. Chứng minh rằng với mọi số nguyên a thì luôn tồn tại một dãy vô hạn {ak} sao cho dãy {ak + a} chứa hữu hạn số nguyên tố.

Bài 20. Chứng minh rằng mỗi số tự nhiên ñều biểu diễn ñược dưới dạng hiệu của hai số tự nhiên có cùng số ước nguyên tố.

Bài 21. Cho p là số nguyên tố lẻ và a1, a2, ..., ap - 2 là dãy các số nguyên dương sao cho p không là ước của ak và ak

k - 1 với mọi k = 1, 2, ..., p - 2.

Chứng minh rằng tồn tại một số phần tử trong dãy trên có tich khi chia cho p dư 2.

Bài 22. Chứng minh rằng nếu ước số nguyên tố nhỏ nhất p của số nguyên dương n không

vượt quá 3 n thì p

n là số nguyên tố.

Bài 23. (Balan 2000) Cho dãy các số nguyên tố p1, p2, ... thỏa mãn tính chất: pn là ước nguyên tố lớn nhất của pn - 1 + pn - 2 + 2000. Chứng minh rằng dãy số trên bị chặn.

Bài 24. Cho a1, a2, ..., an là các số tự nhiên ñôi một khác nhau và có ước nguyên tố không lớn hơn 3. Chứng minh rằng

.31

...11

21

<+++naaa

Bài 25. Gọi A là tập các số nguyên tố p sao cho phương trìn

x2 + x + 1 = py

có nghiệm nguyên x, y. Chứng minh rằng A là tập vô hạn.

Bài 26. Chứng minh rằng tồn tại vô số số k nguyên dương sao cho p2 + k là hợp số với mọi số nguyên tố p.

Bài 27. Cho n là số nguyên dương sao cho n2 + n + 1 phân tích ñược thành tích của 4 số nguyên tố. Chứng minh rằng n ≥ 67.

Bài 28. Chứng minh rằng không thể phân tích bất kì số nguyên tố nào thành tổng bình phương của hai sô tự nhiên theo hai cách khác nhau.

Bài 29. Tìm ba số nguyên tố p, q, r sao cho p2 + q2 + r2 cũng là số nguyên tố.

Bài 30. Tìm các số nguyên tố p sao cho 211p - 2 chia hết cho 11p.

Page 26: Lời nói ñầu - lovetoan.files.wordpress.com · Chứng minh rằng nếu a3 + b3 + c3 chia hết cho 9 thì một trong ba số a, b, c phải chia hết cho 3. Lời giải

Nguyễn Văn Thảo Chuyên ñề Số Học - Phần I

26

Chương II

CÁC BÀI TOÁN VỀ ðỒNG DƯ

II.1. ðịnh nghĩa và các tính chất cơ bản của ñồng dư

II.1.1. Lí thuyết

II.1.1.1 ðịnh nghĩa

Cho ba số nguyên a, b, m (m ≠ 0). Ta nói a ñồng dư với b theo modulo n nếu a – b chia hết cho m.

Kí hiệu: a ≡ b (mod m).

II.1.1.2 Các tính chất

a) a ≡ b (mod m) ⇔ a – b ⋮ m

b) Nếu ai ≡ bi (mod m) với mọi i = 1, 2, …, n thì 1 1

n n

i ii i

a b= =

≡∑ ∑ (mod m).

c) Nếu a ≡ b (mod m) và c ≡ d (mod m)

thì a – c ≡ b – d (mod m).

d) a ≡ b (mod m) và b ≡ c (mod m) ⇒ a ≡ c (mod m).

e) Nếu ai ≡ bi (mod m) với mọi i = 1, 2, …, n thì 1 1

n n

i ii i

a b= −

≡∏ ∏ (mod m)

f) Nếu a ≡ b (mod m) thì an ≡ bn (mod m).

g) Cho P(x) là ña thức tuỳ ý với hệ số nguyên

nếu a ≡ b (mod m) thì P(a) ≡ P(b) (mod m)

Chứng minh

a) Hiển nhiên

b) Do ai ≡ bi (mod m) nên ta có ai – bi ⋮ m

Mà 1 1 1

( )n n n

i i i ii i i

a b a b= = =

− = −∑ ∑ ∑ ⋮ m

Từ ñó suy ra ñpcm.

c) Do a ≡ b (mod m) và c ≡ d (mod m)

nên a –b và c – d ñều chia hết cho m

Mà (a - c) – (b - d) = (a - b) – (c - d) ≡ o (mod m)

Suy ra ñpcm.

Page 27: Lời nói ñầu - lovetoan.files.wordpress.com · Chứng minh rằng nếu a3 + b3 + c3 chia hết cho 9 thì một trong ba số a, b, c phải chia hết cho 3. Lời giải

Nguyễn Văn Thảo Chuyên ñề Số Học - Phần I

27

d) Giả sử a = mq + r

Vì a ≡ b (mod m) nên b = mp + r

Vì b ≡ c (mod m) nên c = ml + r

⇒ a ≡ b (mod m).

(Các phần còn lại xin nhường bạn ñọc)

Page 28: Lời nói ñầu - lovetoan.files.wordpress.com · Chứng minh rằng nếu a3 + b3 + c3 chia hết cho 9 thì một trong ba số a, b, c phải chia hết cho 3. Lời giải

Nguyễn Văn Thảo Chuyên ñề Số Học - Phần I

28

II.1.2. Các ví dụ

Ví dụ 1. A và B là hai số có 7 chữ số khác nhau từ 1 ñến 7 và A > B.

Chứng minh rằng A không chia hết cho B.

Lời giải

Kí hiệu S(n) là tổng các chữ số của n.

Khi ñó S(A) = S(B) = 1 + 2 + … + 7 = 28.

Mà A ≡ S(A) (mod 9); B ≡ S(B) (mod 9).

Từ ñó suy ra A ≡ B ≡ 1 (mod 9).

Giả sử A ⋮ B hay A = pB, p nguyên dương.

Do A >B nên p > 1.

Vì A và B có 7 chữ số khác nhau từ 1 ñến 7 nên

1111111 < B < A < 7777777

Suy ra

7A

B<

Từ ñó suy ra 1 < p < 7

Mà B ≡ 1 (mod 9) nên B = 9k + 1 ⇒ A = p(9k + 1) ≡ p (mod 9)

Mặt khác A ≡ 1 (mod 9) nên p ≡ 1 (mod 9). Vô lí vì 1 < p < 7.

Vậy ñiều giả sử là sai.

Suy ra A không chia hết cho B.

Ví dụ 2. Chứng minh rằng 103n+1 không thể biểu diễn thành tổng hai lập phương.

Lời giải

Ta thấy với mọi số nguyên a thì

3

3

3

0(mod7)

1(mod 7)

1(mod 7)

a

a

a

≡ ≡ −

Do ñó với mọi a, b nguyên thì

3 3

3 3

3 3

0(mod 7)

1(mod 7)

2(mod7)

a b

a b

a b

+ ≡

+ ≡ ± + ≡ ±

Mà 103n+1 ≡ 3.33n ≡ 3.27n ≡ 3(-1)n ≡ ± 3 (mod7)

Nên suy ra ñpcm.

Page 29: Lời nói ñầu - lovetoan.files.wordpress.com · Chứng minh rằng nếu a3 + b3 + c3 chia hết cho 9 thì một trong ba số a, b, c phải chia hết cho 3. Lời giải

Nguyễn Văn Thảo Chuyên ñề Số Học - Phần I

29

Ví dụ 3. Cho 11 số nguyên dương a1, a2, …, a11. Chứng minh rằng luôn tồn tại các số xi ∈ {-1, 0, 1} , i = 1, 2, …, 11 không ñồng thời bằng 0 sao cho:

x1a1 + x2a2 + … + x11a11 chia hết cho 2047.

Lời giải

ðặt S = {b1a1 + b2a2 + … + b11a11| bi ∈ {0, 1}}.

Khi ñó |S| = 2048

Do ñó khi chia các số của S cho 2047 phải có 2048 số dư.

Vì vậy phải tồn tại hai số trong S mà khi chia cho 2047 có cùng số dư

Giả sử hai số ñó là 11 11

1 1

,m i i n i ii i

A b a A c a= =

= =∑ ∑ .

Ta có Am – An ⋮ 2047.

Mà 11 11

1 1

( )m n i i i i ii i

A A b c a x a= =

− = − =∑ ∑

Vì bi, ci ∈ {0, 1} nên xi ∈ {-1, 0, 1}

Lại có (b1, b2, …, b11) ≠ (c1, c2, …, c11) nên xi không ñồng thời bằng 0.

ðpcm.

Ví dụ 4. Xét 100 số tự nhiên liên tiếp 1, 2, …, 100.

Gọi A là số thu ñược bằng cách xếp một cách tuỳ ý 100 số ñó thành một dãy, B là số thu ñược bằng cách ñặt một cách tuỳ ý các dấu cộng vào giữa các chữ số của A. Chứng minh rằng cả A và B ñều không chia hết cho 2010.

Lời giải

Kí hiệu S(n) là tổng các chữ số của số tự nhiên n.

Ta thấy từ 1 ñến 100 xuất hiện 21 chữ số 1, 20 chữ số 2, 3, 4, 5, 6, 7, 8, 9.

Do ñó S(A) = 21.1 + 20(2 + 3 + … + 9) = 901

Mà A ≡ S(A) ≡ 1 (mod 3)

Nên A không chia hết cho 3 do ñó A không chia hết cho 2010.

Giả sử sau khi ñã ñặt các dấu cộng vào giữa các chữ số của A ta ñược

B = b1 + b2 + … + bn.

Khi ñó

B = b1 + b2 + … + bn ≡ S(b1) + S(b2) + … + S(bn) (mod 3).

Mà S(b1) + S(b2) + … + S(bn) = S(A) ≡ 1 (mod 3)

Suy ra B ≡ 1 (mod 3). Do ñó B không chia hết cho 2010.

Page 30: Lời nói ñầu - lovetoan.files.wordpress.com · Chứng minh rằng nếu a3 + b3 + c3 chia hết cho 9 thì một trong ba số a, b, c phải chia hết cho 3. Lời giải

Nguyễn Văn Thảo Chuyên ñề Số Học - Phần I

30

Ví dụ 5. Chứng minh rằng 11n+2 + 122n+1 chia hết cho 133 với mọi số tự nhiên n.

Lời giải

Ta có 11n+2 + 122n+1 = 121.11n + 12.144n

= 133.11n + 12(144n – 11n)

Mà 144n – 11n ⋮ 144-11 = 133

Nên ta có ñpcm.

Ví dụ 6. Chứng minh rằng 19.8n + 17 là hợp số với mọi số tự nhiên n.

Lời giải

Ta có

19.8n + 17 ≡ 19.(-1)n + 17 (mod 3)

Nếu n chẵn thì

19.(-1)n + 17 ≡ 19 + 17 ≡ 0 (mod 3)

Mà 19.8n + 17 > 3 nên 19.8n + 17 là hợp số khi n chẵn.

Nếu n = 4k + 1

Ta có

19.8n + 17 = 19.84k + 1 + 17

≡ 48.642k + 17 (mod 13)

≡ 48.(-1)2k + 17 (mod 12)

≡ 0 (mod 13)

Do ñó 19.8n + 17 là hợp số.

Nếu n = 4k + 3 thì

19.84k+3 + 17 ≡ 0 (mod 5)

Tóm lại với mọi n thì 19.8n + 17 là hợp số.

Ví dụ 7.(Nga 2000) Tìm các số nguyên tố p và q sao cho p + q = (p - q)3.

Lời giải

Ta có

(p - q)3 = p + q ≠ 0

Nên p và q là hai số nguyên tố phân biệt.

Ta có

p – q ≡ 2p (mod p + q)

⇒ (p - q)3 ≡ 8p3 (mod p + q)

⇒ 8p3 ≡ 0 (mod p + q). (*)

Page 31: Lời nói ñầu - lovetoan.files.wordpress.com · Chứng minh rằng nếu a3 + b3 + c3 chia hết cho 9 thì một trong ba số a, b, c phải chia hết cho 3. Lời giải

Nguyễn Văn Thảo Chuyên ñề Số Học - Phần I

31

Do (p, q) = 1 ⇒ (p, p + q) = 1

⇒ (p3, p + q) = 1

Từ (*) ⇒ 8 ⋮ p + q

⇒ p + q ≤ 8

Mà p, q ≥ 2 nên 2≤ q < p ≤ 5

Do ñó (p, q) = (5, 3).

Ví dụ 8. Cho a là một số nguyên dương lẻ. Chứng minh rằng

2 2 2 22 , 2n n m m

a a+ +

nguyên tố cùng nhau với moi số nguyên dương m ≠ n.

Lời giải

Giả sử m > n

Gọi p là một ước nguyên tố của 2 22n n

a +

Ta có

2 22 (mod )n n

a p≡ − (*)

Bình phương hai vế của (*) m – n lần ta ñược:

2 22 (mod )m m

a p≡

Do ñó 2 2 22 2.2 (mod )m m m

a p+ ≡ nên p không thể là ước của 2 22m m

a + .

Từ ñó suy ra ñiều phải chứng minh.

Page 32: Lời nói ñầu - lovetoan.files.wordpress.com · Chứng minh rằng nếu a3 + b3 + c3 chia hết cho 9 thì một trong ba số a, b, c phải chia hết cho 3. Lời giải

Nguyễn Văn Thảo Chuyên ñề Số Học - Phần I

32

II.1.3. Bài tập

Bài 1. Chứng minh rằng với mọi n nguyên dương ta có:

a) 124 22 ++nn

≡ 0 (mod 7)

b) )9(mod011522 ≡−+ nn.

Bài 2. Tìm các số tự nhiên x, y, z thỏa mãn

2x.3y + 1 = 17z.

Bài 3. Tìm tất cả các số nguyên dương n sao cho 5n ≡ - 1 (mod 72000).

Bài 4. Chứng minh rằng với mọi n lẻ thì

1n + 2n + ... + nn ⋮ 1 + 2 +... + n.

Bài 5. Cho p là số nguyên tố. a, b là hai số nguyên bất kì. Chứng minh rằng

(a + b)p ≡ ap + bp (mod p).

Bài 6. Cho a, n nguyên dương, p nguyên tố thỏa mãn

2p + 3p = an

Chứng minh rằng n = 1.

Bài 7. Tìm x nguyên sao cho |x| ≤ 1997 sao cho x2 + (x + 1)2 ⋮ 1997.

Bài 8. Tìm số nguyên dương n = 2p3q sao cho n + 25 là một số chính phương.

Bài 9. Cho các số nguyên không âm a1 , a2 < ... < a101 < 5050. Chứng minh rằng tồn tại bốn số nguyên phân biệt sao cho

(ak + al - am - an) ⋮ 5050.

Bài 10. Chứng minh rằng tồn taị số nguyên dương a sao cho mọi số nguyên k thì các số m = k2 + k - 3a và n = k2 + k + 1 - 3a ñều không chia hết cho 20002001

.

Bài 11. Chứng minh rằng tồn tại một số tự nhiên 2001 chữ số gồm toàn chữ số 1, 2 và chia hết cho 22001.

Bài 12. Chứng minh rằng từ 11 số tự nhiên tuỳ ý luôn chon ñược ra hai số sao cho hiệu bình phương của chúng chia hết cho 20.

Page 33: Lời nói ñầu - lovetoan.files.wordpress.com · Chứng minh rằng nếu a3 + b3 + c3 chia hết cho 9 thì một trong ba số a, b, c phải chia hết cho 3. Lời giải

Nguyễn Văn Thảo Chuyên ñề Số Học - Phần I

33

II.2. ðịnh lý Fermat, ñịnh lý euler và ñịnh lý Wilson

II.2. 1. Hệ thặng dư ñầy ñủ và hệ thặng dư thu gọn.

II.2.1.1 ðịnh nghĩa 1

Nếu x ≡ y (mod m) ta nói y là một thặng dư của x modulo m.

Tập S = {x1, x2, …, xm} ñược gọi là một hệ thặng dư ñầy ñủ modulo m nếu mỗi số nguyên y tuỳ ý ñều tồn tại duy nhất một số xi sao cho y ≡ xi (mod m).

II.2.1.2 Các tính chất cơ bản

+) Tập {1, 2, …, m - 1} là một hệ thặng dư ñầy ñủ modulo m.

+) Mọi hệ thặng dư ñầy ñủ modulo m ñều có ñúng m phần tử.

+) Một hệ gồm m phần tử là hệ thặng dư ñầy ñủ modulo m khi và chỉ khi hai phần tử khác nhau bất kì của nó không ñồng dư với nhau modulo m.

+) Mỗi số nguyên m luôn có vô số hệ thặng dư ñầy ñủ.

+) Với mỗi số nguyên a, m > 0. Tập tất cả các số x nguyên thoả mãn x ≡ a (mod m) lập thành một cấp số cộng. Tập hợp này ñựpc gọi là một lớp thặng dư modulo m.

+) Với mỗi số nguyên dương m thì luôn có m lớp thặng dư modulo m.

II.2.1.3 ðịnh lí 1

Cho a, b, m là các số nguyên. Khi ñó a ≡ b (mod m) thì (a, m) = (b, m).

Chứng minh

Ta có a ≡ b (mod m) ⇒ a – b ≡ 0 (mod m) ⇒ a = b + mq

⇒ (a, m) = (b + mq, m) = (b, m).

ðpcm.

II.2.1.4 ðịnh nghĩa 2

Tập S = {x1, x2, …, xn} với các số xi phân biệt gọi là một hệ thặng dư thu gọn modulo m nếu (xi, m) = 1 với mọi i = 1, 2, …, n và mọi số nguyên y nguyên tố cùng nhau với m ñều tồn tại số xi sao cho y ≡ xi (mod m).

Nhận xét:

+) Ta có thể thu ñược một hệ thặng dư thu gọn bằng cách loại ra khỏi hệ thặng dư ñầy ñủ những số không nguyên tố cùng nhau với m.

+) Mọi hệ thặng dư ñầy ñủ ñều có cùng số phần tử, số phần tử của một hệ thặng dư thu gọn kí hiệu là ϕ(m). ϕ(m) gọi là phi hàm Euler.

+) Nêu p là số nguyên tố thì ϕ(p) = p -1.

+) ϕ(m) bằng số các số nguyên không vượt quá m và nguyên tố cùng nhau với m.

Page 34: Lời nói ñầu - lovetoan.files.wordpress.com · Chứng minh rằng nếu a3 + b3 + c3 chia hết cho 9 thì một trong ba số a, b, c phải chia hết cho 3. Lời giải

Nguyễn Văn Thảo Chuyên ñề Số Học - Phần I

34

II.2.1.5 ðịnh lí 2

Cho (a, m) = 1. Nếu S = {x1, x2, …, xn l} là một hệ thặng dư thu gọn (hoặc ñầy ñủ) modulo m thì aS = {ax1, ax2, …, axn } cũng là một hệ thặng dư thu gọn ( hoặc tương ứng ñầy ñủ) modulo m.

Chứng minh

Ta có (a, m) = 1 ⇒ Nếu axi ≡ axj (mod m) thì xi ≡ xj (mod m)

Do ñó/ với i ≠ j thì xi không ñồng dư với xj ⇒ axi cũng không ñồng dư với axj (mod m).

Suy ra các phần tử của của aS ñôi một phân biệt theo modulo m.

Mà S và aS có cùng số phần tử do ñó nếu S là hệ thặng dư ñầy ñủ thì aS cũng là hệ thặng dư ñầy ñủ.

Nếu S là hệ thặng dư thu gọn thì ta chỉ cần chứng minh các phần tử của aS ñều nguyên tố cùng nhau với m.

Thật vậy, vì (a, m) = 1 và (xi, m) = 1 với mọi i ⇒ (axi, m) = 1 ⇒ aS là hệ thặng dư thu gọn modulo m.

II.2.1.6 ðịnh lí Eurler

Cho a, m là các số nguyên thoả mãn (a, m) = 1. Khi ñó ( ) 1(mod )ma mϕ ≡

Chứng minh

Gọi y1, y2, …, yϕ(m) là môt hệ thặng dư thu gọn modulo m.

Vì (a, m) = 1 nên ay1, ay2, …, ayϕ(m) cũng là một hệ thặng dư thu gọn modulo m.

Do ñó, với mọi i ∈ {1, 2, …, ϕ(m)} ñều tồn tại duy nhất j ∈ {1, 2, …, ϕ(m)} sao cho yi ≡ ayj (mod m).

Từ ñó ta có ( ) ( ) ( )

( )

1 1 1

( ) (mod )m m m

mi i i

i i i

y ay a y mϕ ϕ ϕ

ϕ

= = =

≡ =∏ ∏ ∏

Mà (yi, m) = 1 với mọi i = 1, 2, …, ϕ (m)

Suy ra aϕ (m) ≡ 1 (mod m).

II.2.1.7 ðịnh lí Fermat

Cho p là một số nguyên tố, a là một số nguyên bất kì không chia hết cho p. Khi ñó

ap-1 ≡ 1 (mod p).

Chứng minh

Page 35: Lời nói ñầu - lovetoan.files.wordpress.com · Chứng minh rằng nếu a3 + b3 + c3 chia hết cho 9 thì một trong ba số a, b, c phải chia hết cho 3. Lời giải

Nguyễn Văn Thảo Chuyên ñề Số Học - Phần I

35

Do a không chia hết cho p nên (a, p) = 1. Do ñó, theo ñịnh lí Euler ta có

aϕ (p) ≡ 1 (mod p)

Mà p là số nguyên tố nên ϕ(p) = p – 1 ⇒ ap – 1 ≡ 1 (mod p)

ñpcm.

+) Nhận xét

Từ ñịnh lí Fermat suy ra với mọi số nguyên a và số nguyên tố p thì

ap ≡ a (mod p).

II.2.1.7 ðịnh lí Wilson

Cho p là một số nguyên tố. Khi ñó

(p - 1)! ≡ -1 (mod p)

(Phần chứng minh của ñịnh lí khá ñơn giản, xin nhường cho bạn ñọc )

Page 36: Lời nói ñầu - lovetoan.files.wordpress.com · Chứng minh rằng nếu a3 + b3 + c3 chia hết cho 9 thì một trong ba số a, b, c phải chia hết cho 3. Lời giải

Nguyễn Văn Thảo Chuyên ñề Số Học - Phần I

36

II.2.2. Các ví dụ

Ví dụ 1. Cho n là một số tự nhiên bất kì. Chứng minh rằng

n7 – n ≡ 0 (mod 42).

Lời giải

Do 7 là một số nguyên tố nên, theo ñịnh lí Fermat, ta có

n7 – n ≡ 0 (mod 7). (1)

Ta lại có

n7 – n = n(n6 - 1) = (n - 1)n(n + 1)(n4 + n2 + 1) ≡ 0 (mod 6) (2)

Mà (6, 7) = 1 nên từ (1) và (2) suy ra ñpcm.

Ví dụ 2. Cho p là một số nguyên tố lớn hơn 7. Chứng minh rằng

(3p – 2p - 1) ⋮ 42p.

Lời giải

Ta có

3p – 2p – 1 = (3p - 3) – (2p - 2) ≡ 0 (mod p). (1)

Mặt khác

3p – 2p – 1 = (3p - 1) – 2p ⋮ 2 (2)

Mà p > 7 ⇒ p lẻ

Do ñó

3p – 2p – 1 ≡ - (-1)p – 1 ≡ 0 (mod 3) (3)

Bây giờ ta cần chứng minh 3p – 2p – 1 ⋮ 7

Ta có

3p – 2p - 1 = 3.3p-1 – 2p – 1

=1 1

2 23.9 2 1 3.2 2 1p p

p p− −

− − ≡ − − (mod 7)

= 1 1

2 22 2 2 1p p

p+ −

+ − −

Do (p, 3) = 1 nên p = 3k + 1 hoặc p = 3k + 2

Nếu p = 3k + 1 thì 1 1

2 22 2 2 1p p

p+ −

+ − − = 1 1

2 28 1 2 2 2 2 (mod 7)p p

k p p+ +

− + − ≡ −

1 1 1

2 2 22 (2 1) 2 (8 1) 0(mod 7)p p p

k+ − +

= − = − ≡

Page 37: Lời nói ñầu - lovetoan.files.wordpress.com · Chứng minh rằng nếu a3 + b3 + c3 chia hết cho 9 thì một trong ba số a, b, c phải chia hết cho 3. Lời giải

Nguyễn Văn Thảo Chuyên ñề Số Học - Phần I

37

Nếu p = 3k + 2 ta chứng minh tương tự.

Từ ñó suy ra ñpcm.

Ví dụ 3. Cho x là một số nguyên tố. Khi ñó, phương trình x2 ≡ -1 (mod p) (1) có nghiệm khi và chỉ khi p = 2 hoặc p ≡ 1 ( mod 4).

Lời giải

Nếu p = 2, phương trình có nghiệm x = 1.

Nếu p ≡ 1 (mod 4) ⇒ 1

2

p − là số chẵn

Khi ñó ta sẽ chứng minh x = 1

( )!2

p − là một nghiệm của (1).

Thật vậy, ta có

1 ( 1)(mod )

2 ( 2)(mod )

...

1 1( )(mod )

2 2

p p

p p

p pp

≡ − − ≡ − − − + ≡ −

Do ñó 1

21 1

( )! ( 1) ( 1)( 1)...2 2

pp pp p

−− +≡ − − − (mod p)

12 2

1

2

1 1 1( !) ( 1) .1.2...( ). ...( 1)(mod )

2 2 2

( 1) ( 1)!

p

p

p p pp p

p

− − +⇒ ≡ − −

= − −

Mà theo ñịnh lí Wilson ta có (p - 1)! ≡ -1 (mod p)

p ≡ 1 (mod 4) nên 1

2

p − là số chẵn

Từ ñó suy ra 1

2( 1) ( 1)! 1(mod )p

p p−

− − ≡ −

Do ñó x = 1

!2

p − là một nghiệm của (1).

Ngược lại, nếu p là số nguyên tố lẻ và (1) có nghiệm

Khi ñó, gọi a là một nghiệm của (1)

Ta có

Page 38: Lời nói ñầu - lovetoan.files.wordpress.com · Chứng minh rằng nếu a3 + b3 + c3 chia hết cho 9 thì một trong ba số a, b, c phải chia hết cho 3. Lời giải

Nguyễn Văn Thảo Chuyên ñề Số Học - Phần I

38

1 11 2 2 2( ) ( 1) (mod )

p ppa a p

− −− = ≡ −

Mà theo ñịnh lí Fermat, ta có ap-1 ≡ 1 (mod p) nên 1

21 ( 1) (mod )p

p−

≡ −

suy ra 1

2

p − chẵn hay p ≡ 1 (mod 4)

ñpcm.

Ví dụ 4. Cho a, b là hai số nguyên thỏa mãn 24a2 + 1 = b2. Chứng minh rằng có một và chỉ một trong hai số ñó chia hết cho 5.

Lời giải

Ta có 24a2 – b2 = 1 không chia hết cho 5 nên a và b không thể cùng chia hết cho 5.

Giả sử a và b cùng không chia hết cho 5

Theo ñịnh lí Fermat ta có

a4 – 1 ≡ 0 (mod 5)

b4 – 1 ≡ 0 (mod 5)

Do ñó

(a2 – b2)(a2 + b2) = a4 – b4 ≡ 0 (mod 5)

Nếu a2 + b2 ≡ 0 (mod 5) thì 25a2 + 1 = a2 + b2 ≡ 0 (mod 5) (vô lí)

Vậy a2 – b2 ≡ 0 (mod 5)

⇒ 23a2 + 1 = b2 – a2 ≡ 0 (mod 5) ⇒ 23a2 + 1 ≡ 0 (mod 5)

Vì (a, 5) = 1 nên a ≡ ± 1 (mod 5) hoặc a ≡ ± 2 (mod 5)

Nếu a ≡ ± 1 (mod 5) thì

0 ≡ 23a2 + 1 ≡ 23(± 1)2 + 1 ≡ -1 (mod 5) (vô lí)

Nếu a ≡ ± 2 (mod 5) thì

0 ≡ 23a2 + 1 ≡ 23(± 2)2 + 1 ≡ 3 (mod 5) (vô lí).

Vậy ñiều giả sử là sai

Từ ñó ta có ñpcm.

Ví dụ 5. Cho p là một số nguyên tố, a và b là hai số nguyên dương.Chứng minh rằng:

abp – bap ≡ 0 (mod p).

Lời giải

Ta có

abp – bap = ab(bp-1 – ap-1 )

Page 39: Lời nói ñầu - lovetoan.files.wordpress.com · Chứng minh rằng nếu a3 + b3 + c3 chia hết cho 9 thì một trong ba số a, b, c phải chia hết cho 3. Lời giải

Nguyễn Văn Thảo Chuyên ñề Số Học - Phần I

39

Nếu ab ⋮ p thì hiển nhiên abp – bap ⋮ p.

Nếu ab không chia hết cho p ⇒ (a, p) = (b, p) = 1

⇒ ap-1 – 1 ≡ 0 (mod p) và bp-1 – 1 ≡ 0 (mod p)

⇒ ap-1 – bp-1 ≡ 0 (mod p)

⇒ abp – bap ≡ 0 (mod p)

ñpcm.

Ví dụ 6. Cho a là một số nguyên. Chứng minh rằng a2 + 1 không có ước nguyên tố dạng 4k + 3. Từ ñó suy ra các phương trình sau không có nghiệm nguyên dương.

a) 4xy – x – y = z2

b) x2 – y3 = 7.

Lời giải

Giả sử a2 + 1 có ước nguyên tố p = 4k + 3 ⇒ (a, p) = 1.

Khi ñó

ap-1 + 1 = a4k+ 2 + 1 = (a2)2k + 1 + 1 ⋮ a2 + 1 (1)

Mặt khác, theo ñịnh lí Fermat, ta có

ap – 1 – 1 ⋮ p (2)

Từ (1) và (2) suy ra 2 ⋮ p ⇒ p = 2 (vô lí vì 2 không có dạng 4k + 3)

Vậy a2 + 1 không có ước nguyên tố dạng 4k + 3.

Áp dụng

a) Ta có

4xy – x – y = z2

⇔ (4x - 1)(4y - 1) = 4z2 + 1

⇔ (4x - 1)(4y - 1) = (2z)2 + 1

Do 4x – 1 ≥ 3 với mọi x nguyên dương và có dạng 4k + 3 nên nó có ít nhất một ước nguyên tố dạng 4k + 3

Mà (2z)2 + 1 không có ước nguyên tố dạng 4k + 3 nên phương trình trên vô nghiệm.

b) Ta có

x2 – y3 = 7 ⇔ x2 + 1 = y3 + 8

⇔ x2 + 1 = (y + 2)(y2 – 2y + 4)

Nếu y chẵn thì

(y + 2)(y2 – 2y + 4) ⋮ 4

Page 40: Lời nói ñầu - lovetoan.files.wordpress.com · Chứng minh rằng nếu a3 + b3 + c3 chia hết cho 9 thì một trong ba số a, b, c phải chia hết cho 3. Lời giải

Nguyễn Văn Thảo Chuyên ñề Số Học - Phần I

40

⇒ x2 + 1 ⋮ 4 ⇒ x2 ≡ -1 (mod 4) vô lí

Do ñó y lẻ ⇒ y = 4k + 1 hoặc y = 4k + 3

Nếu y = 4k + 1

⇒ y + 2 = 4k + 3 nên có có ít nhất một ước nguyên tố dạng 4k + 3.

Mà x2 + 1 không có ước nguyên tố dạng 4k + 3 ⇒ phương trình trên không có nghiệm trong trường hợp này.

Nếu y = 4k + 3 ⇒ y2 – 2y + 4 ≡ (-1)2 – 2(-1) (mod 4) ≡ 3 (mod 4)

nên phương trình trên cũng vô nghiệm trong trường hợp này.

Từ ñó ta có ñpcm.

Ví dụ 7. Cho a, b là các số nguyên. p là một số nguyên tố có dạng 4k + 3. Chứng minh rằng nếu x2 + y2 ⋮ p thì x ⋮ p và y ⋮ p. Từ ñó suy ra phương trình sau vô nghiệm nguyên:

x2 + 2y + 4y2 = 37

Lời giải

Giả sử p = 4k + 3 là số nguyên tố thỏa mãn x2 + y2 ⋮ p nhưng x không chia hết cho p ⇒ x2 không chia hết cho p ⇒ y không chia hết cho p.

Theo ñịnh lí Fermat ta có

xp – 1 – 1 ⋮ p ⇒ (x2)2k + 1 – 1 ⋮ p

Tương tự, ta cung có

(y2)2k + 1 – 1 ⋮ p

suy ra

(x2)2k + 1 + (y2)2k + 1 ≡ 2 (mod p)

Mà (x2)2k + 1 + (y2)2k + 1 ⋮ x2 + y2 ⋮ p ⇒ 2 ⋮ p ⇒ p = 2 (vô lí)

Vậy x ⋮ p và y ⋮ p.

Áp dụng

Ta có

x2 + 2x + 4y2 = 37

⇔ (x +1)2 + (2y)2 = 38 ⋮ 19 = 4.4 + 3

Do ñó x + 1 ⋮ 19 và 2y ⋮ 19.

Vì x + 1 và 2y không thể cùng bằng 0 nên |x + 1| ≥ 19 hoặc |2y| ≥ 19

Khi ñó (x + 1)2 + (2y)2 ≥ 192 > 38 nên phương trình trên vô nghiệm.

Ví dụ 8. Cho p ≥ 7, p nguyên tố. Chứng minh rằng số S = 11…1 (p – 1 chữ số 1) chia hết cho p.

Page 41: Lời nói ñầu - lovetoan.files.wordpress.com · Chứng minh rằng nếu a3 + b3 + c3 chia hết cho 9 thì một trong ba số a, b, c phải chia hết cho 3. Lời giải

Nguyễn Văn Thảo Chuyên ñề Số Học - Phần I

41

Lời giải

Ta có

S = 110 1

9

p− −

Vì p ≥ 7 nên (10, p) = 1

Do ñó

10p – 1 – 1 ⋮ p

10p – 1 – 1 ⋮ (10 - ) = 9

Mà (p, 9) = 1 nên 10p – 1 – 1 ⋮ 9p

Từ ñó suy ra S ⋮ p.

ñpcm.

Ví dụ 9. [IMO - 2005] Cho dãy số (an) xác ñịnh như sau

an = 2n + 3n + 6n – 1 với n = 1, 2, …

Tìm số tự nhiên nguyên tố cùng nhau với mọi số hạng của dãy trên.

Lời giải

Ta sẽ chứng minh rằng với mọi số nguyên tố p ñều tồn tại một số hạng an chia hết cho p.

Thật vậy, ta có a2 = 22 + 32 + 62 – 1 = 48 chia hết cho 2 và 3.

Xét p ≥ 5

Ta có

(2, p) = 1; (3, p) = 1; (6, p) = 1

Do ñó, từ ñịnh lí Fermat suy ra

2p – 1 ≡ 3p – 1 ≡ 6p – 1 ≡ 1 (mod p)

Từ ñó dễ dàng chứng minh ñược 6ap – 2 ⋮ p

Mà (p, 6) = 1 nên ap – 2 ⋮ p

Do ñó chỉ có số 1 là số tự nhiên duy nhất nguyên tố cùng nhau với mọi số hạng của dãy (an).

Ví dụ 10. [IMO - 2003] Tìm số nguyên dương k nhỏ nhất sao cho tồn tại các số x1, x2, …, xk sao cho:

x13 + x2

3 + … + xk3 = 20022002.

Lời giải

Ta có 2002 ≡ 4 (mod 9) ⇒ 20023 ≡ 43 ≡ 1 (mod 9)

20022002 = (20023)667.2002 ≡ 2002 (mod 9) ≡ 4 (mod 9)

Page 42: Lời nói ñầu - lovetoan.files.wordpress.com · Chứng minh rằng nếu a3 + b3 + c3 chia hết cho 9 thì một trong ba số a, b, c phải chia hết cho 3. Lời giải

Nguyễn Văn Thảo Chuyên ñề Số Học - Phần I

42

Mặt khác, với mọi số nguyên a ta có

a3 ≡ ± 1 (mod 9) hoặc a3 ≡ 0 (mod 9)

Do ñó

x13 ; x1

3 + x23; x1

3 + x23 + x3

3 không thể ñồng dư với 4 modulo 9 ñược.

Tức là với k ≤ 3 thì phương trình trên không có nghiệm nguyên.

Ta sẽ chứng minh k = 4 là giá trị cần tìm.

Thật vậy, ta có 2002 = 103 + 103 + 13 + 13

Mà 2002 = 3. 667 + 1

⇒ 20022002 = 2002. (2002667)3

= (103 + 103 + 13 + 13 ) (2002667)3

= (10.2002667)3 + (10.2002667)3 + (2002667)3 + (2002667)3

Vậy với k = 4 thì phương trình trên có nghiệm.

KL: k = 4 là giá trị cần tìm.

Ví dụ 11. (Balan - 98) Cho dãy số (an) ñược xác ñịnh như sau:

a1 = 1, 1[ ]2

n n na a a−= + với mọi n ≥ 2.

Chứng minh rằng dãy (an) có vô số số hạng chia hết cho 7.

Lời giải

Giả sử trong dãy trên có hữu hạn số hạng chia hết cho 7.

Khi ñó gọi ak là số hạng cuối cùng của dãy chia hết cho 7.

ðiều này là tồn tại vì có a5 = 7 ⋮ 7.

Ta có

a2k = a2k-1 + ak; a2k + 1 = a2k + ak.

Do ñó

a2k + 1 ≡ a2k ≡ a2k – 1 ≡ x (mod 7).

Với x ⋮ 7.

Mặt khác

a4k – 2 = a4k – 3 + a2k – 1 ≡ a4k – 3 + x (mod 7)

a4k – 1 = a4k – 2 + a2k – 1 ≡ a4k – 2 + x (mod 7) ≡ a4k – 3 + 2x (mod 7)

a4k = a4k – 1 + a2k ≡ a4k – 3 + 3x (mod 7)

a4k + 1 = a4k + a2k ≡ a4k – 3 + 4x (mod 7)

a4k + 2 = a4k + 1 + a2k + 1 ≡ a4k – 3 + 5x (mod 7).

Page 43: Lời nói ñầu - lovetoan.files.wordpress.com · Chứng minh rằng nếu a3 + b3 + c3 chia hết cho 9 thì một trong ba số a, b, c phải chia hết cho 3. Lời giải

Nguyễn Văn Thảo Chuyên ñề Số Học - Phần I

43

a4k + 3 = a4k + 2 + a2k + 1 ≡ a4k – 3 + 6x (mod 7)

Ta có a4k – 3 ⋮ 7 và x ⋮ 7 nên trong các số a4k – 3 + ix (i = 1, 2, …, 6.) phải có số chia hết cho 7 nên trong các số a4k – 3 + i , i = 1, 2, …, 6 phải có số chia hết cho 7.

Vậy ñiều giả sử là sai

Từ ñó ta có ñiều phải chứng minh.

Ví dụ 12. Chứng minh rằng nếu p là một số nguyên tố thì (p - 2)! – 1 ⋮ p.

Nếu p > 5 thì (p - 2)! – 1 không là luỹ thừa của p.

Lời giải

Theo ñịnh lí Wilson ta có

(p - 1)! ≡ - 1 (mod p) ≡ p – 1 (mod p)

⇔ (p - 1)(p - 2)! ≡ p - 1 (mod p) (*)

Do (p, p - 1) = 1 nên

(*) ⇔ (p - 2)! ≡ 1 (mod p)

Với p > 5, giả sử (p - 2)! – 1 = pn

Ta có

(p - 2)! ⋮ p – 1 ⇒ pn + 1 ⋮ p – 1

pn + 1 = (pn – 1) + 2 ≡ 2 (mod p - 1)

⇒ 2 ≡ 0 (mod p – 1)

Vô lí vì p > 5

Vậy có ñiều phải chứng minh.

Ví dụ 13.

a) Cho a là một số nguyên dương. Chứng minh rằng mọi ước nguyên tố p của a2 + 1 với p > 2 thì p ñều có dạng 4k + 1.

b) Chứng minh rằng có vô số số nguyên tố có dạng 4k + 1.

Lời giải

1) Giả sử p là số nguyên tố dạng 4k + 3 và p | a2 + 1

Khi ñó

a2 ≡ -1 (mod p) ⇒ 1

2 2 2 12( ) ( )p

ka a−

+= ≡ -1 (mod p)

Mặt khác theo ñịnh lí Fermat, ta có

Page 44: Lời nói ñầu - lovetoan.files.wordpress.com · Chứng minh rằng nếu a3 + b3 + c3 chia hết cho 9 thì một trong ba số a, b, c phải chia hết cho 3. Lời giải

Nguyễn Văn Thảo Chuyên ñề Số Học - Phần I

44

ap – 1 ≡ 1 (mod p)

Do ñó

2 ≡ 0 (mod p)

Vô lí vì p > 2

Vậy có ñiều phải chứng minh

b) Theo phần a) ta có mọi ước nguyên tố của (n!)2 + 1 ñều có dạng 4k + 1

Giả sử có hữu hạn ước nguyên tố dạng 4k + 1 khi ñó gọi p là số nguyên tố lớn nhất có dạng 4k + 1.

Ta có mọi ước nguyên tố lớn hơn 2 của (p!)2 + 1 ñều có dạng 4k + 1 và ñều không lớn hơn p (do p lớn nhất dạng 4k + 1)

((p!)2 + 1, q) = 1 với mọi q ≤ p dó ñó (p!)2 + 1 không có ước nguyên tố nhỏ hơn hoặc bằng p (vô lí)

Vậy ta có ñiều phải chứng minh.

Ví dụ 14. Chứng minh rằng với mọi số nguyên tố p, tồn tại vô hạn các số nguyên dương n sao cho

2n – n ⋮ p (*)

Lời giải

Nếu p = 2 thì mọi n chẵn ñều thỏa mãn (*)

Nếu p > 2, theo ñịnh lí Fermat, ta có

2p – 1 ≡ 1 (mod p)

⇔ 2m(p - 1) ≡ 1 mod p

Chọn m = kp -1, n = m(p - 1) = (kp - 1)(p - 1) ≡ 1 (mod p)

Khi ñó

2n - n = 2m(p - 1) – n ≡ 1 – 1 ≡ 0 (mod p)

Từ ñó suy ra ñpcm.

Ví dụ 15. (Bulgarian – 95) Tìm số các số tự nhiên n > 1 sao cho

a25 – a ≡ 0 (mod n)

Với mọi số tự nhiên a.

Lời giải

Với mọi số nguyên tố p thì

(p2, p25 - p) = p ⇒ p25 – p ⋮ p

Page 45: Lời nói ñầu - lovetoan.files.wordpress.com · Chứng minh rằng nếu a3 + b3 + c3 chia hết cho 9 thì một trong ba số a, b, c phải chia hết cho 3. Lời giải

Nguyễn Văn Thảo Chuyên ñề Số Học - Phần I

45

Do ñó n ⋮ p2 với mọi p nguyên tố.

suy ra n là tích của các số nguyên tố phân biệt

(Số tự nhiên như vậy ñược gọi là số squarefree)

Mặt khác

225 – 2 = 2.32.5.7.13.17.241

Nhưng n không thể chia hết cho 17 và 241 vì

325 – 3 ≡ -6 (mod 17)

và 325 – 3 ≡ 29 (mod 241)

Bây giờ ta xét p = 2, 3, 5, 7, 13.

+) p = 2, ta có với mọi a nguyên thì a25 – a ⋮ 2.

+) p = 3. Nếu a ⋮ 3 thì a25 – a ⋮ 3

Nếu a ⋮ 3 ⇒ a2 ≡ 1 (mod 3) ⇒ a25 = (a2)12. a ≡ a (mod 3)

⇒ a25 – a ⋮ 3.

Tương tự cho p = 5, 7, 13 thì a25 – a ⋮ p với mọi a nguyên.

Do ñó n chính là tích của k (1 ≤ k ≤ 5) số trong 5 số trên

Từ ñó suy ra có 25 – 1 = 31 số tự nhiên như vậy

Ví dụ 16. Cho n 5≥ là số tự nhiên .Chứng minh rằng

−n

n )!1( ⋮ n-1 .

Lời giải

a) Trường hợp 1. n là số nguyên tố

Theo ñịnh lý Winson (n-1)!≡ -1(mod n) suy ra ((n-1)!+1 ⋮n

Ta có

11)!1(11)!1()!1(−

+−=

−+−

=

−n

n

nn

n

n

n (vì 1

10 <<

n)

= 1)-(n )1()!1(⋮

n

nn −−− vì (n, n - 1) = 1

b) Trường hợp 2. n là hợp số

+) n không là bình phương của một số nguyên tố.

Khi ñó n = rs với 1< r < s < n.

Do (n,n-1)=1 suy ra s < n-1 )n(n-) rs(n-)! (n- 111 =⇒ ⋮ hay (n-1)! = kn(n-1)

suy ra

Page 46: Lời nói ñầu - lovetoan.files.wordpress.com · Chứng minh rằng nếu a3 + b3 + c3 chia hết cho 9 thì một trong ba số a, b, c phải chia hết cho 3. Lời giải

Nguyễn Văn Thảo Chuyên ñề Số Học - Phần I

46

)(n)k(nn

)!(n11

1−−=

−⋮ .

+) n = p 2 với p là một số nguyên tố.

Do p =2 n, n 5≥ suy ra p 1233 2 +>≥⇒≥ pp p

12 2 −<⇒ pp hay 2p < n-1

Nên 1 < p < 2p < n-1

Suy ra (n-1)! ⋮ p.2p.(n-1) = 2n(n-1).

Từ ño suy ra ) (n-n

)!(n1

1⋮

−.

Vậy ta cố ñiều phải chứng minh.

Page 47: Lời nói ñầu - lovetoan.files.wordpress.com · Chứng minh rằng nếu a3 + b3 + c3 chia hết cho 9 thì một trong ba số a, b, c phải chia hết cho 3. Lời giải

Nguyễn Văn Thảo Chuyên ñề Số Học - Phần I

47

II.2.3. Bài tập

Bài 1.

a) Cho a là số nguyên sao cho (a, 7) = 1. Chứng minh rằng

a12 - 1 ⋮ 7.

b) a là số nguyên dương sao cho (a, 240) = 1. Chứng minh rằng

a4 - 1 ⋮ 240.

Bài 2. Cho a1 + a2 + ... + an ⋮ 30, và a1, a2, ..., an nguyên. Chứng minh rằng

.30... 552

51 ⋮naaa +++

Bài 3. Chứng minh rằng

n7 - n ⋮ 42

với mọi số nguyên n.

Bài 4. Cho n nguyên dương. Chứng minh rằng

a) ⋮32143 +

+n 11.

b) 1921102 +

+n

⋮ 23.

c) 2622

+n

≡ 16 (mod 37).

Bài 5. Cho p là số nguyên tố lớn hơn 17. Chứng minh rằng

P16 ≡ 1 (mod 16320).

Bài 6. Cho p là số nguyên tố lẻ. Chứng minh rằng

2(p-3)! ≡ -1 (mod p).

Bài 7. Cho n là hợp số, n ≠ 4. Chứng minh rằng (n - 1)! ≡ 0 (mod n).

Bài 8. Cho p và q là hai số nguyên tố phân biệt. Chứng minh rằng

qp - 1 + pq - 1 ≡ 1 (mod pq).

Bài 9. Cho a, b là các số nguyên, p là số nguyên tố. Chứng minh rằng

(a + b)p ≡ ap + bp (mod p).

Bài 10. Chứng minh rằng n và n + 2 là cặp số nguyên tố sinh ñôi khi và chỉ khi

4[(n - 1)! + 1] + n ≡ 0 (mod n(n + 2)).

Bài 11. Cho p là một số nguyên tố lẻ. Chứng minh rằng số m =8

19 −p

là một hợp số lẻ,

không chia hết cho 3 và 3m - 1 ≡ 1 (mod m).

Bài 12. Chứng minh rằng với mọi số nguyên tố p, tồn tại vô hạn số nguyên dương n thỏa mãn

2n - n ⋮ p.

Page 48: Lời nói ñầu - lovetoan.files.wordpress.com · Chứng minh rằng nếu a3 + b3 + c3 chia hết cho 9 thì một trong ba số a, b, c phải chia hết cho 3. Lời giải

Nguyễn Văn Thảo Chuyên ñề Số Học - Phần I

48

Bài 13. Tìm tất cả các số nguyên tố p sao cho

p)(p mod1522 ≡

Bài 14. Cho a, b là hai số nguyên dương sao cho 2a - 1, 2b - 1 và a + b ñều là các số nguyên tố. Chứng minh rằng aa + bb và ab + ba ñều không chia hết cho a + b.

Bài 15. Tìm số nguyên dương n sao cho n = a2 + b2 với a, b là hai số nguyên dương

nguyên tố cùng nhau và ab chia hết cho mọi số nguyên tố nhỏ hơn hoặc bằng n .

Bài 16. Cho p là một số nguyên tố lẻ. Chứng minh rằng không tồn tại x, y nguyên thỏa mãn hệ thức

xp + yp = p[(p - 1)!]p.

Bài 17. Tìm ba số nguyên tố p, q, r sao cho p2 + q2 + r2 cũng là số nguyên tố.

Page 49: Lời nói ñầu - lovetoan.files.wordpress.com · Chứng minh rằng nếu a3 + b3 + c3 chia hết cho 9 thì một trong ba số a, b, c phải chia hết cho 3. Lời giải

Nguyễn Văn Thảo Chuyên ñề Số Học - Phần I

49

II.3. Số chính phương mod p

II.3.1. Lí thuyết

II.3.1.1 ðịnh nghĩa 1

Cho số nguyên tố p. Số nguyên a ñược gọi là số chính phương (mod p) nếu tồn tại số nguyên x sao cho x2 ≡ a ( mod p).

Nhận xét:

+) Mọi số chính phương ñều là số chính phương (mod p)

+) a ≡ 0 (mod p) thì a2 ≡ a (mod p) nên mọi a ≡ 0 (mod p) ñều là số chính phương (mod p). Do ñó, từ ñây về sau, ta chỉ xét số nguyên a sao cho (a, p) = 1.

+) Mọi số nguyên lẻ ñều là số chính phương (mod 2)

II.3.1.2 Kí hiệu Legendre

Cho p là số nguyên tố lẻ.

1a

p

=

nếu a là số chính phương mod p

a

p

= -1 nếu a không là số chính phương mod p.

Kí hiệu trên gọi là kí hiệu Legendre.

II.3.1.3 ðịnh lí 1

Cho p là một số nguyên tố lẻ. Khi ñó 1

21 1(mod )pa

a pp

− = ⇔ ≡

(1)

1

21 1(mod )pa

a pp

− = − ⇔ ≡ −

(2)

Chứng minh

Giả sử a là số chính phương mod p, khi ñó, tồn tại số tự nhiên x sao cho

x2 ≡ a (mod p)

Do (a, p) = 1 nên (x, p) = 1.

Theo ñịnh lí Fermat ta có 1

1 21 (mod )p

px a p−

−≡ ≡

Ngược lại, nếu có (1) thì với mỗi k ∈ {1, 2, …, p - 1} có duy nhất một số k’ ∈ {1, 2, …, p - 1} sao cho k.k’ ≡ a (mod p)

Page 50: Lời nói ñầu - lovetoan.files.wordpress.com · Chứng minh rằng nếu a3 + b3 + c3 chia hết cho 9 thì một trong ba số a, b, c phải chia hết cho 3. Lời giải

Nguyễn Văn Thảo Chuyên ñề Số Học - Phần I

50

Nếu tồn tại k = k’ thì k.k’ = k2 ≡ a (mod p) ⇒ a là số chính phương (mod p).

Trái lại, tập {1, 2, …, p - 1} ñược chia thành 1

2

k − tập con {k, k’} rời nhau sao cho k.k’ ≡

a (mod p). ⇒ (p - 1)! ≡ 1

2 1(mod )p

a p−

Mặt khác, theo ñịnh lí Wilson, ta có

(p - 1)! ≡ -1 (mod p)

Từ ñó suy ra

1 ≡ -1 (mod p) ⇒ p = 2 (vô lí vì p lẻ)

Vậy (1) ñược chứng minh xong.

Theo ñịnh lí Fermat, ta có ap – 1 ≡ 1 (mod p)

1

2

1

2

1(mod )

1(mod )

p

p

a p

a p

≡ −

Do ñó, (2) ñược chứng minh.

II.3.1.4 ðịnh lí 2 (Bổ ñề Gauss)

Cho p là số nguyên tố lẻ, a là số nguyên, (a, p) = 1. Xét tập

{ka| k = 1, 2, …, 1

2

p −}.

Gọi rk ≡ ka (mod p), 1 ≤ rk ≤ p.

Gọi n là số các số rk thuộc khoảng ( ;2

pp ). Khi ñó

1

2 ( 1) (mod )p

na p−

≡ − .

(n chính là số bội số của a trong khoảng ( ;2

pp ))

Chứng minh

Ta có

ka ≡ rk (mod p)

Cho k chạy từ 1 ñến (p - 1)/2 rồi nhân các ñẳng thức ñó lại ta ñược: 1

1 22

1

1! (mod )

2

pp

kk

pa r p

−−

=

−≡∏

Page 51: Lời nói ñầu - lovetoan.files.wordpress.com · Chứng minh rằng nếu a3 + b3 + c3 chia hết cho 9 thì một trong ba số a, b, c phải chia hết cho 3. Lời giải

Nguyễn Văn Thảo Chuyên ñề Số Học - Phần I

51

Gọi A = |2k k

pr r >

; B = |

2k k

pr r <

Khi ñó 1

2

1

( 1) ( ) (mod )k k k k

p

nk k k k k

k r A r B r A r B

r r r p r r p

= ∈ ∈ ∈ ∈

= ≡ − −∏ ∏ ∏ ∏ ∏

Ta có với mọi rk ∈ A thì p – rk ≤ 1

2

p −.

Mặt khác Nếu ri ∈ A, rj ∈ B thì p – ri ≠ rj vì nếu p – ri = rj thì ri + rj = p

⇒ a(i + j) ≡ 0 (mod p) ⇒ i + j ≡ 0 (mod p).

ðiều này không thể vì i và j thuộc 1

(1; )2

p − nên 1 < i + j < p .

Từ ñó suy ra

11 2

2

1

1 1! ( 1) !

2 2

pp

nk

k

p pa r

−−

=

− −≡ ≡ −∏ (*)

Mà 1

( !, ) 12

pp

−= nên từ (*) suy ra

1

2 ( 1) (mod )p

na p−

≡ −

ñpcm.

Từ ñó ta có một số hệ quả quan trọng sau.

Hệ quả 1. Cho p là số nguyên tố lẻ. Gọi n là số các số chẵn thuộc ( ;2

pp ) thì ta có

1

22 ( 1) (mod )p

n p−

≡ − .

Chứng minh

Từ ñịnh lí 2, cho a = 2 ta có ñiều phải chứng minh.

Hệ quả 2. Cho p là số nguyên tố lẻ. Khi ñó 1

21

( ) ( 1)p

p

−−= −

Chứng minh

(Hiển nhiên)

Hệ quả 3. Cho p là số nguyên tố lẻ. Khi ñó

2( ) 1

p= ⇔ p ≡ ± 1 (mod 8)

Page 52: Lời nói ñầu - lovetoan.files.wordpress.com · Chứng minh rằng nếu a3 + b3 + c3 chia hết cho 9 thì một trong ba số a, b, c phải chia hết cho 3. Lời giải

Nguyễn Văn Thảo Chuyên ñề Số Học - Phần I

52

2 1

82( 1)

p

p

− = −

Chứng minh

Giả sử p = 4k + 1, tập các số chẵn trong khoảng (0; p) là {2i | 1 ≤ i ≤ 2k}

Khi ñó, số các số chẵn trong khoảng ( ;2

pp ) là n = k.

Tương tự như vậy, nếu p = 4k – 1 thì ta cũng tính ñược số các số chẵn trong khoảng

( ;2

pp ) là n = k.

Do ñó, theo hệ quả 1, ta có 1

22 ( 1) (mod )p

n p−

≡ − ≡ (-1)k (mod p)

2 là số chính phương mod p ⇔ 1

21 2 ( 1) (mod )p

k p−

≡ ≡ −

⇔ k chẵn ⇔ p ≡ ± 1 (mod p).

2 không là số chính phương mod p ⇔ k lẻ ⇔ p ≡ ± 3 (mod p)

ñpcm.

Hệ quả 4. Cho p là số nguyên tố lẻ. Gọi n là số các bội của 3 trong khoảng ( ;2

pp ).

Khi ñó

1

23 ( 1) (mod )p

n p−

≡ −

Chứng minh

ðược suy ra hiển nhiên từ ñịnh lí, trong trường hợp a = 3.

Hệ quả 5. Cho p là số nguyên tố có dạng 6k ± 1. Khi ñó, 3 là số chính phương mod p khi và chỉ khi p ≡ ± 1 (mod 12).

Chứng minh

Giả sử p = 6k + 1. Tập hợp các số là bội của 3 trong khoảng (0; p) là

{3i | 1 ≤ i ≤ 2k}

3i > 1

12 6 6

p pi k i k⇔ > = + ⇔ ≥ +

Page 53: Lời nói ñầu - lovetoan.files.wordpress.com · Chứng minh rằng nếu a3 + b3 + c3 chia hết cho 9 thì một trong ba số a, b, c phải chia hết cho 3. Lời giải

Nguyễn Văn Thảo Chuyên ñề Số Học - Phần I

53

Do ñó, số các số là bội của 3 trong khoảng ( ;2

pp ) là n = k.

Tương tự khi p = 6k – 1, ta cũng chứng minh ñược n = k.

Vậy trong cả hai trường hợp ta ñều chứng minh ñược số các số là bội của 3 trong khoảng

( ;2

pp ) là n = k.

Vì vậy, theo hệ quả 4, ta có 1

23 ( 1) (mod )p

n p−

≡ − ≡ (-1)k (mod p)

⇒ 3

( ) 1p

= ⇔ (-1)k ≡ 1 (mod p)

⇔ k chẵn

⇔ p ≡ ± 1 (mod 12).

ñpcm.

II.3.1.5 ðịnh lí 2

Cho p là số nguyên tố. a ≡ b (mod p), (a, p) = (b, p) = 1. Khi ñó

( ) ( )a b

p p= .

Chứng minh

Do a ≡ b (mod p) ⇒ 1 1

2 2 (mod )p p

a b p− −

≡ ⇒ ( ) ( )a b

p p=

ñpcm.

II.3.1.5 ðịnh lí 4

Cho p là một số nguyên tố. a1, a2, …, an là các số nguyên không chia hết cho p. Khi ñó

1 2 1 2...( ) ( )( )...( )n na a a aa a

p p p p=

Chứng minh

(Phần chứng minh khá ñơn giản, xin nhường cho bạn ñọc)

II.3.1.6 ðịnh lí 5

Cho p là số nguyên tố lẻ, a là số nguyên không chia hết cho p. Khi ñó

Page 54: Lời nói ñầu - lovetoan.files.wordpress.com · Chứng minh rằng nếu a3 + b3 + c3 chia hết cho 9 thì một trong ba số a, b, c phải chia hết cho 3. Lời giải

Nguyễn Văn Thảo Chuyên ñề Số Học - Phần I

54

1

2 ( 1) (mod )p

sa p−

≡ −

Với s =

1

2

1

p

k

ka

p

=

Chứng minh

Gọi n là số các bội của a trong khoảng ( ;2

pp ), theo ñịnh lí 2, ta có

1

2 ( 1) (mod )p

na p−

≡ −

Vì vậy, ta chỉ cần chứng minh s – n là số chẵn.

(Phần này khá ñơn giản, xin nhường lại cho bạn ñọc).

II.3.1.6 ðịnh lí 7 (Luật tương hỗ Gauss)

Cho p, q là hai số nguyên tố lẻ phân biệt.

Khi ñó:

a) Nếu có ít nhất một trong hai số có dạng 4k + 1 thì

( ) ( )p q

q p= .

b) Nếu cả hai số có dạng 4k + 3 thì

( ) ( ).p q

q p= −

Chứng minh

a) ðặt

s1 =

1 1

2 2

21 1

;

p q

i i

iq ips

p q

− −

= =

=

∑ ∑

Khi ñó

s1 + s2 = 1 1

( )( )2 2

p q− −

(tính chất này ñã ñược chứng minh trong tài liệu “Một số bài toán về phần nguyên ” của cùng tác giả).

Do p hoăc q có dạng 4k + 1 nên 1 1

( )( )2 2

p q− − là số chẵn.

Nên s1 và s2 có cùng tính chẵn lẻ.

Page 55: Lời nói ñầu - lovetoan.files.wordpress.com · Chứng minh rằng nếu a3 + b3 + c3 chia hết cho 9 thì một trong ba số a, b, c phải chia hết cho 3. Lời giải

Nguyễn Văn Thảo Chuyên ñề Số Học - Phần I

55

Theo ñịnh lí 5, ta có

1

1

2 ( 1) (mod )p

sq p−

≡ −

22 ( 1) (mod )q

sp q≡ −

Từ ñó dễ dàng suy ra ñiều phải chứng minh.

b) Chứng minh tương tự.

II.3.2. Các ví dụ

Ví dụ 1. Cho p là số nguyên tố có dạng 8k + 5 hoặc 8k + 7. x, y là hai số nguyên thỏa mãn x2 + 2y2 chia hết cho p.

Chứng minh rằng x ⋮ p và y ⋮ p.

Lời giải

Giả sử ngược lại x ⋮ p ⇒ 2y2 ⋮ p ⇒ y ⋮ p.

x2 + 2y2 ⋮ p ⇒ x2 ≡ -2y2 (mod p)

⇒ 2 22

( ) ( )x y

p p

−=

⇒ 1 = 22 2 1 2

( )( ) ( ) ( )( )y

p p p p p

− − −= = . (1)

Nếu p = 8k + 5 ⇒ 2

1

2

1

8

1( ) ( 1) 1

2( ) ( 1) 1

p

p

p

p

−= − =

= − = −

Thay vào (1) ñược 1 = -1 (vô lí).

Nếu p = 8k + 1 ⇒ 2

1

2

1

8

1( ) ( 1) 1

2( ) ( 1) 1

p

p

p

p

−= − = −

= − =

cũng không thoả mãn (1)

Vậy ñiều giả sử là sai

Từ ñó suy ra ñpcm.

Page 56: Lời nói ñầu - lovetoan.files.wordpress.com · Chứng minh rằng nếu a3 + b3 + c3 chia hết cho 9 thì một trong ba số a, b, c phải chia hết cho 3. Lời giải

Nguyễn Văn Thảo Chuyên ñề Số Học - Phần I

56

Ví dụ 2. Cho 22 1n

k = + với n nguyên dương. Chứng minh rằng k là số nguyên tố khi và

chỉ khi k là ước của 1

23 1k−

+ .

Lời giải

Nếu k là ước của 1

23 1k−

+ thì ta có

1

23k−

≡ -1 (mod k) (1)

⇔ 3k -1 ≡ 1 (mod k) (2)

Gọi d là bậc của 3 modulo k

Từ (1) và (2) ta có d | k – 1 nhưng d lại không chia hết 1

2

k −

⇒ d = k – 1 ⇒ k là số nguyên tố.

Ngược lại, k là số nguyên tố

Ta có k là số nguyên tố dạng 4l + 1 nên theo luật tương hỗ Gauss ta có

3( ) ( )

3

k

k=

Mà k ≡ 2 (mod 3) nên 2

( ) ( ) 13 3

k= = − (do 2 không phải số chính phương mod 3).

Từ ñó suy ra 1 1

2 23 1(mod ) 3 1 0(mod )k k

k k− −

≡ − ⇔ + ≡ .

ñpcm.

Ví dụ 3. Chứng minh rằng với mọi n nguyên dương, số 2n + 1 không có ước nguyên tố dạng 8k + 7.

Lời giải

Giả sử tồn tại số nguyên tố p = 8k + 7 sao cho p | 2n + 1

Nếu n chẵn, ta có

2n ≡ -1 (mod p)

Suy ra -1 là số chính phương mod p

Do ñó 1

4 321

1 ( ) ( 1) ( 1) 1p

k

p

−+−

= = − = − = − (vô lí)

Nếu n lẻ, ta có

Page 57: Lời nói ñầu - lovetoan.files.wordpress.com · Chứng minh rằng nếu a3 + b3 + c3 chia hết cho 9 thì một trong ba số a, b, c phải chia hết cho 3. Lời giải

Nguyễn Văn Thảo Chuyên ñề Số Học - Phần I

57

2n ≡ -1 (mod p) ⇒ 2n + 1 ≡ -2 (mod p)

Suy ra -2 là số chính phương mod p

Do ñó, ta có

2 1 21 ( ) ( )( ) 1

p p p

− −= = = − .1 = -1. (vô lí)

Vậy ta có ñiều phải chứng minh.

Ví dụ 4. Tìm x, n nguyên dương sao cho

x3 + 2x + 1 = 2n. (1)

Lời giải

Do x nguyên dương nên x3 + 2x + 1 ≥ 4 ⇒ n ≥ 2.

Nếu n = 2, ta dễ dàng tìm ñược x = 1.

Xét n ≥ 3

(1) ⇔ x(x2 + 2) = 2n – 1 là số lẻ nên x lẻ

Do ñó, x2 + 2 ⋮ 3 ⇒ 2n ≡ (1 mod 3) ⇔ (-1)n ≡ 1 (mod 3)

Từ ñó suy ra n chẵn.

Mặt khác

x3 + 2x + 1 = 2n

⇔ x3 + 2x + 3 = 2n + 2

⇔ (x + 1)(x2 –x + 3) = 2n + 2.

Giả sử p là một ước nguyên tố của x2 – x + 3 ⇒ p lẻ

Khi ñó

2n + 2 ≡ 0 (mod p) ⇔ 2n ≡ -2 (mod p)

Mà n chẵn suy ra 2

( ) 1p

−=

Ta có

1 = 2 11

822 1 2

( ) ( )( ) ( 1) ( 1)pp

p p p

−−− −= = − −

Từ ñó suy ra p = 8k + 1 hoặc p = 8k + 3.

Mà n ≥ 3 ⇒ 2n = 8.2n – 3 ≡ 0 (mod 8)

Do ñó ta suy ra

x3 + 2x + 1 ≡ 0 (mod 8).

Page 58: Lời nói ñầu - lovetoan.files.wordpress.com · Chứng minh rằng nếu a3 + b3 + c3 chia hết cho 9 thì một trong ba số a, b, c phải chia hết cho 3. Lời giải

Nguyễn Văn Thảo Chuyên ñề Số Học - Phần I

58

Mà x lẻ ⇒ x = 8k ± 1, x = 8k ± 3

Nếu x = 8k ± 1 ⇒ x3 + 2x + 1 ≡ (± 1)3 + 2(± 1) + 1 ≡ 0 (mod 8) (vô lí)

Nếu x = 8k + 3 thì x3 + 2x + 1 cũng không thể chia hết cho 8.

Nếu x = 8k + 5 thì thỏa mãn.

Khi ñó

x2 – x + 3 ≡ 52 – 5 + 3 ≡ 7 (mod 8)

Hay

x2 – x + 3 = 8l + 7

Mà số dạng 8l + 7 thì không thể có ước nguyên tố dạng 8k + 1 hoặc 8k + 3

Do ñó (1) vô nghiệm khi n ≥ 3.

Vậy x = 1, n = 2.

Ví dụ 5. Cho p là số nguyên tố dạng 12k – 1. Tìm x, y, n nguyên không âm sao cho

3a2 + b2 ⋮ pn (1)

Với a và b không chia hết cho p.

Lời giải

Nếu n = 0 thì mọi a, b nguyên không âm và a ⋮ p, b ⋮ p ñều thỏa mãn (1)

Nếu n ≠ 0 ⇒ a > 0 ⇒ 3a2 + b2 ⋮ p

Do ñó

b2 ≡ -3a2 (mod p)

⇒ 1 1 1

2 22 2 2( ) ( 3) ( ) (mod )p p p

b a p− − −

≡ −

⇒ bp – 1 ≡ 1

2( 3)p−

− ap – 1 (mod p)

(a, p) = (b, p) = 1 ⇒ ap – 1 ≡ bp – 1 ≡ 1 (mod p)

Suy ra 1

2( 3) 1(mod )p

p−

− ≡

Mặt khác 1

6 123 1 3 3 3 3

1 ( ) ( )( ) ( 1) ( ) ( 1) ( ) ( )p

k

p p p p p p

−−− −

= = = − = − = − (*)

Page 59: Lời nói ñầu - lovetoan.files.wordpress.com · Chứng minh rằng nếu a3 + b3 + c3 chia hết cho 9 thì một trong ba số a, b, c phải chia hết cho 3. Lời giải

Nguyễn Văn Thảo Chuyên ñề Số Học - Phần I

59

Lại có p = 12k – 1 nên 3 là số chính phương mod p hay 3

( ) 1p

=

Thay vào (*) ta ñược 1 = -1 (vô lí)

Vậy khi n > 0 thì không tồn tại a, b, n thỏa mãn.

Ví dụ 6. (Serbi - 2008) Giải phương trình

12x + y4 = 2008z

Với x, y, z là các số nguyên không âm.

Lời giải

Nếu z = 0 ⇒ x = y = 0.

Nếu z > 0 ⇒ y > 0 vì 12x ⋮ 251 mà 2008 = 251.8

TH1: x chẵn

Ta có 12x + y4 = 2 2 2 2 22(12 ) ( )x

y a b+ = +

Với a = 12x/2, b = y2.

Vì 2008 = 251.8 ⇒ a2 + b2 ⋮ 251

Mà 251 là số nguyên tố và 251 = 4k + 3 nên cả a và b ñều phải chia hết cho 251

Vô lí vì 12x/2 ⋮ 251.

TH2. x lẻ làm tương tự.

Vậy phương trình có nghiệm duy nhất x = y = z = 0.

Page 60: Lời nói ñầu - lovetoan.files.wordpress.com · Chứng minh rằng nếu a3 + b3 + c3 chia hết cho 9 thì một trong ba số a, b, c phải chia hết cho 3. Lời giải

Nguyễn Văn Thảo Chuyên ñề Số Học - Phần I

60

II.3.3. Bài tập

Bài 1. Chứng minh rằng mọi ước nguyên tố của n4 - n2 + 1 ñều có dạng 12k + 1.

Bài 2. (Ba Lan 2007) Chứng minh rằng phương trình x2 + 5 = y3 không có nghiệm nguyên.

Bài 3. Chứng minh rằng với mỗi số nguyên tố p thì tồn tại các số nguyên a, b sao cho a2 + b2 + 1 chia hết cho p.

Bài 4. Cho số nguyên tố p. Chứng minh rằng 3p + 7p - 4 không là bình phương của một số nguyên.

Bài 5. (IMO 2006) Tìm tất cả các nghiệm nguyên của phương trình

1 + 2x + 22x + 1 = y2.

Bài 6. Cho m, n là các số nguyên sao cho A = m

m n

3

1)3( ++ là một số nguyên. Chứng

minh rằng A là số lẻ.

Bài 7. (ðề nghị IMO 2004)Chứng minh rằng 2n + 1 không có ước nguyên tố dạng 8k + 7.

Chứng minh rằng 123 +n

có ít nhất n ước nguyên tố dạng 8k + 3.

Bài 8. Tìm tất cả các số nguyên dương n sao cho 3n - 1 ⋮ 2n - 1.

Page 61: Lời nói ñầu - lovetoan.files.wordpress.com · Chứng minh rằng nếu a3 + b3 + c3 chia hết cho 9 thì một trong ba số a, b, c phải chia hết cho 3. Lời giải

Nguyễn Văn Thảo Chuyên ñề Số Học - Phần I

61

II. 4. ðịnh lí thặng dư Trung Hoa

II.4.1 Lí thuyết

II.4.1.1. ðịnh lí 1 (ðịnh lí thặng dư Trung Hoa)

Cho hệ phương trình

1 1

2 2

(mod )

(mod )

...

(mod )n n

x r m

x r m

x r m

≡ ≡ ≡

(1)

Trong ñó m1, m2, …, mn là các số nguyên ñôi một nguyên tố cùng nhau. Khi ñó hệ phương trình trên luôn có nghiệm. Nếu x1 và x0 là hai nghiệm của (1) thì x1 ≡ x0 (mod m1m2…mn).

Chứng minh

ðặt si = 1

n

kk

i

m

m=∏

.

Do m1, m2, …, mn ñôi một nguyên tố cùng nhau nên (si, mi) = 1.

Nên với mỗi số nguyên dương si luôn tồn tại một số nguyên hi sao cho

sihi ≡ 1 (mod mi).

ðặt x0 = 1

n

i i ii

s h r=∑

Vì si ⋮ mj với mọi j ≠ i nên

x0 ≡ sihiri (mod mi) ≡ ri (mod mi)

Do ñó x0 chính là một nghiệm của (1).

Giả sử x1 cũng là một nghiệm của (1)

Ta có

x1 ≡ x0 (mod mi) ⇒ x1 – x0 ⋮ mi.với mọi i

Mà m1, m2, …, mn ñôi một nguyên tố cùng nhau nên

x1 – xo ⋮ m1m2…mn.

Từ ñó suy ra ñiều phải chứng minh.

Page 62: Lời nói ñầu - lovetoan.files.wordpress.com · Chứng minh rằng nếu a3 + b3 + c3 chia hết cho 9 thì một trong ba số a, b, c phải chia hết cho 3. Lời giải

Nguyễn Văn Thảo Chuyên ñề Số Học - Phần I

62

II.4.1.2. ðịnh lí 2

Cho hệ phương trình

1 1

2 2

(mod )

(mod )

...

(mod )n n

x r m

x r m

x r m

≡ ≡ ≡

(1)

ðiều kiện cần và ñủ ñể hệ trên có nghiệm là

ri ≡ rj (mod (mi, mj)).

Khi ñó hệ trên có nghiệm duy nhất theo modulo [m1, m2, …, mn].

Chứng minh

(xin nhường cho bạn ñọc)

II.4.1.3. Phương pháp giải hệ (1) trong trường hợp các số mi ñôi một nguyên tố cùng nhau.

Bước 1. ðặt m = m1m2…mn = Mimi với i = 1, 2, …,n.

Bước 2. Tìm các số Ni nghiệm ñúng phương trình

Mix ≡ 1 (mod m)

Bước 3. Tìm ñược một nghiệm của hệ là

x0 = 1

n

i i ii

M N r=∑ là một nghiệm của hệ

Bước 4. Kết luận x = x0 + mt.

Page 63: Lời nói ñầu - lovetoan.files.wordpress.com · Chứng minh rằng nếu a3 + b3 + c3 chia hết cho 9 thì một trong ba số a, b, c phải chia hết cho 3. Lời giải

Nguyễn Văn Thảo Chuyên ñề Số Học - Phần I

63

II.4.2 Các ví dụ

Ví dụ 1. Giải hệ

4(mod11)

3(mod17)

x

x

Lời giải

Xét phương trình

17y ≡ 1(mod 11)

⇔ 6y ≡ 1 (mod 11)

Ta dễ dàng tìm ñược một nghiệm là N1 = 2.

Xét phương trình

11y ≡ 1 (mod 17)

Ta cũng dễ dàng tìm ñược một nghiệm là N2 = 14.

Suy ra một nghiệm là x0 = 17.2 .4 +11.14.3

Từ ñó tìm ñược nghiệm của hệ là x = 17.2.4 + 11.14. 3 + 11.17t.

Ví dụ 2. Giải hệ

1(mod 2)

2(mod3)

3(mod5)

x

x

x

≡ ≡

Lời giải

Xét phương trình

15y ≡ 1 (mod 2)

Phương trình này có một nghiệm là N1 = 1

Xét phương trình

10y ≡ 1 (mod 3)

Cũng dễ dàng tìm ñược một nghiệm là N2 = 1.

Xét phương trình

6y ≡ 1 (mod 5)

Cũng tìm ñược N3 = 1 là một nghiệm.

Vậy x0 = 15.1.1 + 10.1.2 + 6.1.3 = 53 là một nghiệm của hệ

Do ñó hệ phương trình ñã cho có nghiệm là

x ≡ 53 (mod 30) ≡ 23 (mod 30)

Page 64: Lời nói ñầu - lovetoan.files.wordpress.com · Chứng minh rằng nếu a3 + b3 + c3 chia hết cho 9 thì một trong ba số a, b, c phải chia hết cho 3. Lời giải

Nguyễn Văn Thảo Chuyên ñề Số Học - Phần I

64

Ví dụ 3. Giải hệ

0(mod 2)

0(mod3)

1(mod5)

6(mod 7)

x

x

x

x

≡ ≡

≡ ≡

Lời giải

Xét phương trình

42y ≡ 1 (mod 5)

⇔ 2y ≡ 1 (mod 5)

Tìm ñược N3 = 3.

Xét phương trình

30y ≡ 1 (mod 7)

⇔ 2y ≡ 1 (mod 7)

Tìm ñược N3 = 4

Từ ñó suy ra x0 = 42.3.1 + 30.4.6 là một nghiệm của hệ

Vậy hệ có nghiệm là x ≡

Ví dụ 4. Chứng minh rằng với mọi số nguyên dương k tuỳ ý luôn tồn tại k số nguyên liên tiếp toàn hợp số.

Lời giải

Gọi p1 < p2 < ...<pk là các số nguyên tố tuỳ ý. Khi ñó, theo ñịnh lí thặng dư Trung Hoa, luôn tồn tại n sao cho

−≡

−≡

−≡

)(mod

...

)(mod2

)(mod1

2

1

kpkn

pn

pn

Do ñó với mọi i = 1, 2, ..., k thì n + i ⋮ pk.

Do ñó chỉ cần chọn n > pk thì dãy {n + i| i = 1, 2, ..., n} luôn là hợp số.

Ví dụ 5. (VMO - 2008) Cho m = 20072008. Hỏi có tất cả bao nhiêu số tự nhiên n<m sao cho m | n(2n +1)(5n + 2)?

Lời giải

Ta có m = 32008.2232008.

Ta lại có (n, 2n + 1) = 1; (n, 5n + 2) = (n, 2) ≤ 2

Page 65: Lời nói ñầu - lovetoan.files.wordpress.com · Chứng minh rằng nếu a3 + b3 + c3 chia hết cho 9 thì một trong ba số a, b, c phải chia hết cho 3. Lời giải

Nguyễn Văn Thảo Chuyên ñề Số Học - Phần I

65

(2n + 1; 5n + 2) = (2n + 1, n) = 1

Do ñó m | n(2n + 1)(5n +2) khi và chỉ khi xảy ra một trong các trường hợp sau:

1) m | n

2) m | 2n + 1

3) m | 5n + 2

4) 34096 | n và 2232008 | 2n +1

5) 34096 | n và 2332008 | 5n + 2

6) 34096 | 5n + 2 và 2232008 | 2n + 1

7) 34096 | 5n + 2 và 2332008 | n

8) 34096 | 2n + 1 và 2232008 | 5n + 2

9) 34096 | 2n + 1 và 2232008 | n

Trong mỗi trường hợp ấy, theo ñịnh lí thặng dư Trung hoa, có duy nhất một số tự nhiên n (modulo m) thỏa mãn.

Nên có tất cả 9 số tự nhiên thỏa mãn ñề bài.

Ví dụ 6. (ðề nghị IMO 2002) Trong lưới ñiểm nguyên của mặt phẳng tọa ñộ Oxy, một ñiểm có tọa ñộ là các số nguyên A(x, y) ∈ Z2 ñược gọi là nhìn thấy ñược từ O nếu trên ñoạn OA không có ñiểm nào thuộc Z2, trừ O và A. Chứng minh rằng với mọi số tự nhiên n tuỳ ý, luôn tồn tại hình vuông n x n có các ñỉnh nguyên và mọi ñiểm nguyên bên trong và trên biên của hình vuông ñều không nhìn thấy ñược từ O.

Lời giải

Ta có nếu (x, y) = d

thì ñiểm );(d

y

d

xM là ñiểm nguyên thuộc ñoạn OA với A(x; y).

Do ñó, A(x, y) là ñiểm nhìn thấy ñược từ O khi và chỉ khi (x, y) = 1.

Gọi jip , là các số nguyên tố ñôi một khác nhau với 0 ≤ i, j ≤ n

(có (n +1)2 số nguyên tố như vậy)

Xét hai hệ sau:

−≡

−≡

)...(mod

...

)...(mod1

)...(mod0

,1,0,

,11,10,1

,01,00,0

nnnn

n

n

pppnx

pppx

pppx

Page 66: Lời nói ñầu - lovetoan.files.wordpress.com · Chứng minh rằng nếu a3 + b3 + c3 chia hết cho 9 thì một trong ba số a, b, c phải chia hết cho 3. Lời giải

Nguyễn Văn Thảo Chuyên ñề Số Học - Phần I

66

−≡

−≡

)...(mod

...

)...(mod1

)...(mod0

,,1,0

1,1,11,0

0,0,10,0

nnnn

n

n

pppnx

pppx

pppx

Theo ñịnh lí thặng dư Trung Hoa, tồn tại các số tự nhiên y như vậy.

Mà x + i và y + j ñều chia hết cho pi,j

Do ñó, mọi ñiểm trong hình vuông n x n với (n + 1)2 ñiểm nguyên Ai,j(x + i, y + j) trên ñều không nhìn thấy ñược từ O.

Ví dụ 8. (Nordic - 98) Tồn tại hay không một dãy cô hạn các số tự nhiên

{x1, x2, ..., xn, ...} = {1, 2, ..., n, ...}

Sao cho xi ≠ xj với mọi i ≠ j và

x1 + x2 + ... + xk ⋮ k với mọi k = 1, 2, ...

Lời giải

Ta xây dựng một dãy thỏa mãn ñề bài như sau:

Chọn x1 = 1, x2 = 3, x3 = 2.

Giả sử x1, x2, ..., xn là dãy số thỏa mãn

x1 + x2 + ... + xk ⋮ k với mọi k = 1, 2, ..., n.

Gọi m là số nguyên dương bé nhất không nằm trong dãy x1, x2, ..., xn.

Do (n + 1, n + 2) = 1 nên, theo ñịnh lí thặng dư Trung Hoa, tồn tại số nguyên x lớn hơn max {x1, x2, ..., xn} và thỏa mãn

+−−≡

+−≡

)2(mod

)1(mod

nsmx

nsxvới s = x1 + x2 + ... + xn.

Khi ñó, ñặt xn+1 = x, xn + 2 = m

Từ ñó ta ñược dãy x1, x2, ..., xn+1, xn + 2 thoả mãn

x1 + x2 + ... + xn + xn+1 = s + x ⋮ n +1.

x1 + x2 + ... + xn+1 + xn + 2 = s + x + n ⋮ n + 2.

Do ñó

x1 + x2 + ... + xk ⋮ k với mọi k = 1, 2, ..., n + 2

Cứ tiếp tục như vậy ta thu ñược dãy số vô hạn thỏa mãn yêu cầu bài toán.

Page 67: Lời nói ñầu - lovetoan.files.wordpress.com · Chứng minh rằng nếu a3 + b3 + c3 chia hết cho 9 thì một trong ba số a, b, c phải chia hết cho 3. Lời giải

Nguyễn Văn Thảo Chuyên ñề Số Học - Phần I

67

II.4.3. Bài tập

Bài 1. Giải các hệ phương trình sau:

a)

)7(mod3

)11(mod1

x

x

b)

)7(mod5

)5(mod3

)3(mod1

x

x

x

c)

)11(mod37

)8(mod25

)12(mod15

x

x

x

Bài 2. (IMO 1989) Chứng minh rằng với mọi số tự nhiên n, luôn tồn tại n số tự nhiên liên tiếp mà cả n số ñó ñều không phải luỹ thừa của một số nguyên tố.

Bài 3. (Hàn Quốc 1999) Tìm tất cả các số tự nhiên n sao cho 2n - 1 chia hết cho 3 và tồn

tại m ∈ Z sao cho 4m2 + 1 chia hết cho 3

12 −n

.

Bài 4. Chứng minh rằng với mọi số nguyên dương k lớn tuỳ ý ñều tồn tại k số nguyên liên tiếp gồm toàn hợp số.

Bài 5. Cho S = {a1, a2, ..., an} ⊂ Z. Chứng minh rằng tồn tại một số b ∈ Z sao cho tập b.S = {ba1, ba2, ..., ban} mà mọi phần tử của nó ñều là luỹ thứa lớn hơn 1 của một số nguyên.

Bài 6. Cho f(x) là một ña thức với hệ số nguyên. Giả sử có một tập hữu hạn các số nguyên tố {p1, p2,..., pn} mà với mọi n thì ñều tồn tại pi | f(n). Chứng minh rằng tồn tại một số nguyên tố p sao cho với mọi n thì f(n) ⋮ p.

Page 68: Lời nói ñầu - lovetoan.files.wordpress.com · Chứng minh rằng nếu a3 + b3 + c3 chia hết cho 9 thì một trong ba số a, b, c phải chia hết cho 3. Lời giải

Nguyễn Văn Thảo Chuyên ñề Số Học - Phần I

68

Chương III

MỘT SỐ VẤN ðỀ KHÁC

III.1. Luỹ thừa của một số nguyên

III.1.1 Số chính phương

III.1.1.1 ðịnh nghĩa

Số tự nhiên n ñược gọi là một số chính phương nếu tồn tại m nguyên sao cho n = m2.

Nhận xét: Nếu phân tích tiêu chuẩn n = k

kppp ααα ...21

21 thì:

+) n là số chính phương khi và chỉ khi αi chẵn với mọi i = 1, 2, ..., k.

+) n là luỹ thừa s của một số nguyên khi và chỉ khi αi ⋮ s với mọi i = 1, 2, ..., k.

III.1.1.2 Các ví dụ

Ví dụ 1. Tìm tất cả các số chính phương có dạng A = ab1985

Lời giải

Ta có

4452 = 198025 < A < 198916 = 4462

Từ ñó suy ra không có số chính phương nào thỏa mãn yêu cầubài toán.

Ví dụ 2. Chứng minh rằng nếu abc là số nguyên tố thì b2 - 4ac không phải là số chính phương.

Lời giải

Giả sử b2 - 4ac là số chính phương thì

b2 - 4ac = k2, k nguyên dương.

Ta có

acabaabca 440400.4 2 ++=

= 400a2 + 40ab + b2 - (b2 - 4ac)

= (20a + b)2 - k2

= (20a + b + k)(20a + b - k)

Vì abc là số nguyên tố nên c ≠ 0 ⇒ ac > 0

Do ñó

b > k ⇒ 20a + b + k > 20a + b - k > 20a

Từ ñó suy ra

nma

kbakbaabc .

4

)20)(20(=

−+++=

Page 69: Lời nói ñầu - lovetoan.files.wordpress.com · Chứng minh rằng nếu a3 + b3 + c3 chia hết cho 9 thì một trong ba số a, b, c phải chia hết cho 3. Lời giải

Nguyễn Văn Thảo Chuyên ñề Số Học - Phần I

69

Mà 20a + b + k và 20a + b - k ñều lớn hơn 4a nên m và n ñều lớn hơn 1

Nên abc là hợp số, trái với giả thiết

Từ ñó ta có ñiều phải chứng minh.

Ví dụ 3. Chứng minh rằng một số chính phương luôn có số ước số nguyên dương là lẻ và ngược lại một số tự nhiên có số ước nguyên dương là lẻ thì số ñó phải là số chính phương.

Lời giải

a) Nếu A là một số chính phương thì phân tích tiêu chuẩn của A là

nkn

kk pppA 22

221 ...21=

Từ ñó suy ra số ước số nguyên dương của A là

(2k1 + 1)(2k2 + 1)...(2kn + 1) là số lẻ.

b) Ngược lại: Nếu A có số ước số là lẻ

Xét phân tích tiêu chuẩn của A

nkn

kk pppA ...21

21=

Từ ñó suy ra số ước số của A là

(k1 + 1)(k2 + 1)...(kn + 1) là số lẻ

Do ñó, tất cả các nhân tử trên ñều lẻ

Suy ra ki chẵn với mọi i = 1, 2, ..., n

Từ ñó suy ra A là số chính phương.

Ví dụ 4. (Romani - 2004) Tìm tất cả các số không âm n sao cho tồn tại hai số nguyên a và b sao cho

n2 = a + b và n3 = a2 + b2.

Lời giải

Ta có

2(a2 + b2) ≥ (a + b)2 ⇒ 2n3 ≥ n4 ⇔ 0 ≤ n ≤ 2.

Nếu n = 0, chọn a = b = 0

Nếu n = 1, chọn ñược a = 1, b = 0.

Nếu n = 2, chọn ñược a = b = 2.

Vậy n = 0, n = 1, n = 2 là các giá trị cần tìm.

Ví dụ 5. Chứng minh rằng nếu 2n (n ∈ N*) là tổng của hai số chính phương thì n cũng là tổng của hai số chính phương.

Lời giải

Gọi n là số tự nhiên thỏa mãn

Page 70: Lời nói ñầu - lovetoan.files.wordpress.com · Chứng minh rằng nếu a3 + b3 + c3 chia hết cho 9 thì một trong ba số a, b, c phải chia hết cho 3. Lời giải

Nguyễn Văn Thảo Chuyên ñề Số Học - Phần I

70

2n = a2 + b2 với a, b ∈ N. (1)

Từ (1) suy ra a và b cùng tính chẵn lẻ

Do ñó a +b và a - b là số chẵn

ðặt

=−

=+

yba

xba

2

2

Khi ñó a = x + y và b = x - y

Thay vào (1) ta ñược

2n = (x + y)2 + (x - y)2

⇔ n = x2 + y2.

Từ ñó ta ñược ñiều phải chứng minh.

Ví dụ 6. Cho a, b là hai số nguyên dương sao cho a2 + b2 ⋮ ab + 1.

Chứng minh rằng 1

22

++

ab

ba là số chính phương.

Lời giải

ðặt

1

22

++

=ab

bak (1)

Không mất tính tổng quát ta giả sử a ≥ b.

Khi ñó

(1) ⇔ a2 + b2 - kab - k = 0

⇔ a2 - kb.a + b2 - k = 0.(*)

Giả sử rằng k không phải số chính phương. Xét phương trình

x2 - 2bx + b2 - k = 0 (3)

Ta có a là một nghiệm của (3), gọi a1 là một nghiệm còn lại

Khi ñó

a + a1 = kb

Do a, k, b nguyên nên suy ra a1 cũng là số nguyên.

+) Nếu a1 = 0 thì k = b2, trái với ñiều giả sử.

+) Nếu a1 < 0, ta có

a12 - kba1 + b2 = k

Mà a1 nguyên âm nên a12 -kba1 + b2 > -kba1 = (-a1b)k > k (vô lí)

Page 71: Lời nói ñầu - lovetoan.files.wordpress.com · Chứng minh rằng nếu a3 + b3 + c3 chia hết cho 9 thì một trong ba số a, b, c phải chia hết cho 3. Lời giải

Nguyễn Văn Thảo Chuyên ñề Số Học - Phần I

71

+) Do ño a1 > 0 thì

a.a1 = b2 - k ⇒ a1 = aa

kb<

−2

Dặt b1 = b khi ñó a + b > a1 + b1. ta có

a12 - kb1a1 + b1

2 - k = 0 (4)

Lại xuất phát từ (4) suy ra phương trình (3) có nghiệm tự nhiên mới (a2, b2) mà

a2 + b2 < a1 + b1.

Từ ñó suy ra phương trình hai biến (*) có vô hạn nghiệm tự nhiên thỏa mãn

(a1, b1), (a2, b2), ... thỏa mãn a1 + b1 > a2 + b2 > ...(vô lí)

Nên ñiều giả sử là sai.

Vậy ta có ñiều phải chứng minh.

Vídụ 7. Tìm n nguyên dương sao cho

A = 28 + 211 + 2n

là một số chính phương.

Lời giải

Nếu A là số chính phương thì

28 + 211 + 2n = x2 ⇔ 2n = (x - 48)(x + 48)

Với x là một số nguyên dương nào ñó.

Khi ñó

x - 48 = 2s và x + 48 = 2n - s, n > 2s

Từ ñó suy ra

2n - s - 2s = 96 ⇔ 2s(2n - 2s - 1) = 3.25.

Từ ñó suy ra

=

=⇔

=−

=− 12

5

312

52 n

sssn

KL: n = 12.

Ví dụ 8. Chứng minh rằng

A = 1k + 9k + 19k + 2013k

không phải số chính phương với mọi k nguyên dương lẻ.

Lời giải

Với mọi k nguyên dương lẻ, ta có

1k ≡ 1 (mod 4)

Page 72: Lời nói ñầu - lovetoan.files.wordpress.com · Chứng minh rằng nếu a3 + b3 + c3 chia hết cho 9 thì một trong ba số a, b, c phải chia hết cho 3. Lời giải

Nguyễn Văn Thảo Chuyên ñề Số Học - Phần I

72

9k ≡ 1 (mod 4)

19k ≡ -1 (mod 4)

2013k ≡ 1 (mod 4)

Nên A ≡ 2 (mod 4)

Vậy A không thể là số chính phương.

Ví dụ 9. Tìm số tự nhiên n sao cho n - 50 và n + 50 ñều là số chính phương.

Lời giải

Ta có

=+

=−2

2

50

50

bn

an

với a, b nguyên dương và a > b.

Suy ra b2 - a2 = 100 ⇔ (b - a)(b + a) = 22.52.

Do b - a < b + a và chúng có cùng tính chẵn lẻ nên a + b và b - a phải là các số chẵn. Do ñó

=

=⇔

=+

=−

26

24

50

2

b

a

ab

ab

Từ ñó tìm ñược n = 626.

Ví dụ 10. Chứng minh rằng với mọi số nguyên dương n thì số

24

)21217()21217(nn −−+

là một số nguyên và không phải số chính phương.

Lời giải

Ta có 4)12(21217 +=+ và 4)12(21217 −=−

Do ñó

24

)21217()21217(nn −−+

=22

)12()12(.

2

)12()12( 2222 nnnn −−+−++

ðặt

2

)12()12( 22 nn

A−++

= và 22

)12()12( 22 nn

B−−+

=

Sử dụng nhị thức Niutơn ta suy ra

( 2 + 1)2n = x + y 2 và ( 2 - 1)2n = x - y 2

Page 73: Lời nói ñầu - lovetoan.files.wordpress.com · Chứng minh rằng nếu a3 + b3 + c3 chia hết cho 9 thì một trong ba số a, b, c phải chia hết cho 3. Lời giải

Nguyễn Văn Thảo Chuyên ñề Số Học - Phần I

73

Với x, y là các số nguyên dương.

Từ ñó suy ra

xAnn

=−++

=2

)12()12( 22

và yBnn

=−−+

=22

)12()12( 22

Nên A.B là một số nguyên dương.

Mặt khác, ta lại có

A2 - 2B2 = (A - B 2 )(A + B 2 ) = ( 2 +1)2n( 2 - 1)2n = 1

Do ñó A và B nguyên tố cùng nhau.

Vậy ñể chứng minh AB không phải số chính phương ta chỉ cần chứng minh một trong hai số ñó không phải là số chính phương.

Ta có

2

)12()12( 22 nn

A−++

= =[ ]

12

)12()12(2

−−++ nn

2

)12()12( 22 nn

A−++

= =[ ]

12

)12()12(2

+−−+ nn

Tư ñó dễ dàng suy ra ñược A không phải số chính phương

Vậy suy ra ñiều phải chứng minh.

Ví dụ 11. (Polish - 2001) Cho a, b là các số nguyên sao cho với mọi n thì 2na + b ñều là số chính phương. Chứng minh rằng a = 0.

Lời giải

Giả sử a ≠ 0.

Nếu a ≠0, b = 0 thì 21a + b và 22a + b không thể ñồng thời là số chính phương.

Do ñó a và b ñều phải khác 0.

Từ ñó dễ dàng suy ra ñược a và b ñều dương.

Xét hai dãy số (xn) và (yn) như sau

baybax nn

nn +=+= +2222

Dễ thấy (xn) và (yn) là hai dãy số nguyên dương, ñơn ñiệu tăng vô hạn.

Ta có

(xn + yn)(xn - yn) = 3b

Suy ra 3b ⋮ xn + yn với mọi n ⇒ 3b ≥ xn + yn với mọi n (vô lí)

Vậy ñiều giả sử là sai

Từ ñó suy ra ñiều phải chứng minh.

Page 74: Lời nói ñầu - lovetoan.files.wordpress.com · Chứng minh rằng nếu a3 + b3 + c3 chia hết cho 9 thì một trong ba số a, b, c phải chia hết cho 3. Lời giải

Nguyễn Văn Thảo Chuyên ñề Số Học - Phần I

74

Ví dụ 12. Tìm n nguyên dương sao cho

n2 + 3n

là số chính phương.

Lời giải

Giả sử

n2 + 3n = m2

với m là một số nguyên dương nào ñó.

Ta có

(m - n)(m + n) = 3n

Từ ñó suy ra

=+

=−−kn

k

nm

nm

3

3 (*)

Mà m +n > m - n nên n - 2k ≥ 1.

Nếu n - 2k = 1

Từ (*) suy ra 2n = 3n - k - 3k = 3k(3n - 2k - 1) = 2.3k

⇔ 3k = n = 2k + 1 ⇔ k = 0, k = 1.

Từ ñó tìm ñược n = 1, n = 3.

Nếu n - 2k ≥ 2 ⇒ k ≤ n - k - 2

Từ (*) suy ra 2n = 3n - k - 3k ≥ 3n - k - 3n - k - 2 = 8.3n - k - 2

Theo bất ñẳng thức Bernoulli ta có

8.3n - k - 2 = 8.(1 + 2)n - k - 2 ≥ 8[1 + 2(n - k - 2)]

= 16n - 16k - 24.

Do ñó

2n ≥ 16n - 16k - 24 ⇔ 8k + 12 ≥ 7n .

Mà n - 2k ≥ 2 nên n ≥ 2k + 2 ⇒ 7n ≥ 14k + 14

Từ ñó suy ra 8k + 12 ≥ 14k + 14 vô lí vì k ≥ 0.

KL: n = 1, n = 3.

Page 75: Lời nói ñầu - lovetoan.files.wordpress.com · Chứng minh rằng nếu a3 + b3 + c3 chia hết cho 9 thì một trong ba số a, b, c phải chia hết cho 3. Lời giải

Nguyễn Văn Thảo Chuyên ñề Số Học - Phần I

75

III.1.2 Lập phương của một số nguyên

Ví dụ 1. Chứng minh rằng nếu n là lập phương của một số nguyên (n ≠ -1) thì

n2 + 3n + 3 không thể là lập phương của một số nguyên.

Lời giải

n = 0 thì n2 + 3n + 3 = 3 ≠ m3.

Giả sử n2 + 3n + 3 = k3 với k là một số nguyên nào ñó

Vì n là lập phương của một số nguyên nên

n(n2 + 3n + 3) = l3

⇔ (n + 1)3 - 1 = l3

Với n ≠ 0, n ≠ 1, n nguyên thì 0 < 3n2 + 3n < 3n2 + 3n + 1

⇒ n3 < n3 + 3n2 + 3n < n3 + 3n2 + 3n + 1

⇔ n3 < n3 + 3n2 + 3n < (n + 1)3

Do ñó n3 + 3n2 + 3n không thể là lập phương của một số nguyên.

Vậy ñiều giả sử là sai

Do ñó ta có ñiều phải chứng minh.

Ví dụ 2. (Iran - 98)Chứng minh rằng không có số tự nhiên nào có dạng abab trong hệ cơ số 10 là lập phương của một số nguyên. Hãy tìm cơ số b nhỏ nhất saôch trong hệ cơ số b,

có ít nhất một số có dạng abab là lập phương của một số nguyên.

Lời giải

Ta có abab = 101 ab là lập phương của một số nguyên thì

101 | ab (vô lí)

Vậy abab không thể là lập phương của một số nguyên.

Xét trong hệ cơ số b

Ta có

))(1()1( 2)(

2)( bannabnabab nn ++=+=

với a, b < n và n2 + 1 > an + b.

Nếu n2 + 1 không chia hết cho một số chính phương nào thì n2 + 1 = p1p2...pk

Khi ñó an + b phải chia hết cho (p1p2...pk)2 vô lí

Vậy n2 +1 phải chia hết cho một số chính phương.

Thử trực tiếp thấy n = 7 là số nhỏ nhất như vậy (n2 + 1 = 50)

Mặt khác thấy 10002626 )7( = = 103.

Page 76: Lời nói ñầu - lovetoan.files.wordpress.com · Chứng minh rằng nếu a3 + b3 + c3 chia hết cho 9 thì một trong ba số a, b, c phải chia hết cho 3. Lời giải

Nguyễn Văn Thảo Chuyên ñề Số Học - Phần I

76

Do ñó n = 7 chính là số cần tìm.

Vídụ 3. Cho x, y là các số tự nhiên thỏa mãn x2 + y2 + 6 chia hết cho xy.

Chứng minh rằng xy

yx 622 ++ là lập phương của một số tự nhiên.

Lời giải

Vì x2 + y2 + 6 ⋮ xy nên x2+ y2 + 6 = pxy (1)

Gọi (x0, y0) là nghiệm của (1) và thỏa mãn x0 + y0 nhỏ nhất.

Không mất tính tổng quát, giả sử x0 ≤ y0

Xét phương trình

y2 - px0y + x02 + 6 = 0 (2)

Dễ thấy y0 là một nghiêm của (2). Gọi y1 là nghiệm còn lại

Khi ñó, theo Viet ta có

+=

=+

62010

010

xyy

pxyy (3)

Dễ thấy y1 > 0 nên (x0, y1) cũng là một nghiệm nguyên dương của (1)

Do ñó

x0 + y0 ≤ x0 + y1 ⇒ y0 ≤ y1

Nếu x0 = y0 thì từ (1) ta có

20

020

20 6

262

xx

x

xp +=

+= ∈ Z

⇔ x0 = 1 ⇒ p = 8 = 23

Nếu x0 < y0

+) y0 = y1 thì từ (3) suy ra y02 = x0

2 + 6

⇔ (y0 - x0)(y0 + x0) = 6 (4)

Mà VT(4) ≡ 0 (mod 4) còn VP(4) ≡ 2 (mod 4) nên (4) không xảy ra

+) y0 < y1 nên x0 < y0 < y1

Ta có y0 ≥ x0 + 1 ⇒ y1 ≥ y0 + 1 ≥ x0 + 2 ⇒ y0y1 ≥ (x0 + 1)(x0 + 2)

⇒ x02 + 6 ≥ (x0 + 1)(x0

+ 2)

3

40 ≤⇔ x

Vì x0 nguyên dương nên x0 = 1

Do ñó y0.y1 = x2 + 6 = 7

Page 77: Lời nói ñầu - lovetoan.files.wordpress.com · Chứng minh rằng nếu a3 + b3 + c3 chia hết cho 9 thì một trong ba số a, b, c phải chia hết cho 3. Lời giải

Nguyễn Văn Thảo Chuyên ñề Số Học - Phần I

77

Do ñó tìm ñược y0 = 1, y1 = 7 (không thỏa mãn ñiều kiên x0 < y0)

Vậy ta có p = 8 = 23 (ñiều phải chứng minh).

Ví dụ 4. Chứng minh rằng không tồn tại số tự nhiên n sao cho

2n + 1 - 1 và 2n - 1(2n - 1)

ñồng thời là lập phương của các số nguyên.

Lời giải

Giả sử ngược lại, tồn tại n sao cho 2n + 1 - 1 và 2n - 1(2n - 1) ñều là lập phương của các số nguyên.

Khi ñó

2n - 1(2n - 1) = k3

Mà 2n - 1 không chia hết cho 2 nên 2n - 1 cũng phải là lập phương của một số nguyên hay n - 1 = 3m.

Từ ñó suy ra

a3 = 2n +1 - 1 = 23m + 2 - 1 = 4.8m - 1 ≡ 3(mod 7)

Vô lí vì a3 ≡ 0; ± 1 (mod 7)

Vậy ta có ñiều phải chứng minh.

Ví dụ 5. Chứng minh rằng với mọi số nguyên không âm n thì

A = 2n + 3n + 5n + 6n

không thể là lập phương của một số nguyên.

Lời giải

Ta có 26 ≡ 36 ≡ 56 ≡ 66 ≡ 1 (mod 7)

Mà n = 6k + r với r = 0, 1, 2, 3, 4, 5

Nếu r = 6k thì

A ≡ 4 (mod 7) do ñó A ≠ m3.

Nếu n = 6k + 1 thì

A = 2.(26)k + 3.(36)k + 5.(56)k + 6(66)k ≡ 2 + 3 + 5 + 6 ≡ 2 (mod 7)

nên A ≠ m3

Nếu n = 6k + 2 thì A ≡ 22 + 32 + 52 + 62 ≡ 4 (mod 7) nên A ≠ m3

Nếu n = 6k + 3 thì A ≡ 23 + 33 + 53 + 63 ≡ 5 (mod 7) ⇒ A ≠ m3

Nếu n = 6k + 4 thì A ≡ 24 + 34 + 54 + 64 ≡ 2 (mod 7) ⇒ A ≠ m3

Nếu n = 6k + 5 thì A ≡ 25 + 35 + 55 + 65 ≡ 4 (mod 7) ⇒ A ≠ m3.

Vậy ta có ñiều phải chứng minh.

Page 78: Lời nói ñầu - lovetoan.files.wordpress.com · Chứng minh rằng nếu a3 + b3 + c3 chia hết cho 9 thì một trong ba số a, b, c phải chia hết cho 3. Lời giải

Nguyễn Văn Thảo Chuyên ñề Số Học - Phần I

78

III.1.3 Các bài toán biểu diễn một số nguyên thành tổng các luỹ thừa

Ví dụ 1. Chứng minh rằng nếu 3

12 −n là tích của hai số tự nhiên liân tiếp thì n là tổng

bình phương của hai số nguyên liên tiếp.

Lời giải

Giả sử n là số tự nhiên mà

3

12 −n= a(a + 1) (1)

với a là một số tự nhiên nào ñó.

Ta có

(1) ⇔ n2 = 3a2 + 3a + 1

⇒ 4n2 = 12a2 + 12a + 4

⇒ (2n - 1)(2n + 1) = 3(2a + 1)2.

Do 2n - 1 và 2n + 1 là hai số lẻ liên tiếp và (2n - 1, 2n + 1) = 1 nên

=−

=+

=+

=−

(**)12

312

(*)12

312

2

2

2

2

pn

mn

pn

mn

Từ (*) suy ra p2 = 3m2 +2 (vô lí vì số chính phương chia 3 chỉ dư 0 và 1)

Do ñó, chỉ có (**) xảy ra.

Từ (**) suy ra m và p ñều lẻ

ðặt p = 2k + 1 ta ñược

2n = p2 + 1 = (2k +1)2 + 1 = 4k2 + 4k + 2

⇔ n = (k +1)2 + k2.

ðó là ñiều phải chứng minh.

Ví dụ 2. (Nga - 1996) Cho x, y, p, n, k là các số tự nhiên thỏa mãn

xn + yn = pk. (1)

Chứng minh rằng nếu n > 1, n lẻ và p là một số nguyên tố lẻ thì n là một luỹ thừa của p.

Lời giải

ðặt m = (x, y) thì x = ma, y = mb với (a, b) = 1.

Khi ñó

Page 79: Lời nói ñầu - lovetoan.files.wordpress.com · Chứng minh rằng nếu a3 + b3 + c3 chia hết cho 9 thì một trong ba số a, b, c phải chia hết cho 3. Lời giải

Nguyễn Văn Thảo Chuyên ñề Số Học - Phần I

79

(1) ⇔ mn(an + bn) = pk. (2)

Do p nguyên tố nên từ (2) suy ra m = pq

Do ñó

(2) ⇔ an + bn = pk - nq.

Mà n lẻ nên

an + bn = (a + b)(an - 1 - an - 2b + ... - abn - 2 + bn - 1) (3)

= (a + b)A

Với A = an - 1 - an - 2b + ... - abn - 2 + bn - 1.

Vì p lẻ nên p > 2 suy ra hai số x và y khác tính chẵn lẻ nên ít nhất một trong hai số a và b phải lớn hơn 1.

Do ñó an + bn > a+ b ⇒ A > 1

A(a + b) = pk - nq > 1

Suy ra A và a + b ñều chia hết cho p

Hay a + b = pr

Từ ñó suy ra

A = an - 1 - an - 2(pr - a) + ... - a(pr - a)n - 2 + (pr - a)n - 1 = nan - 1 + Bp.

Mà A ⋮ p ⇒ nan - 1 ⋮ p

a + b ⋮ p, (a, b) = 1 nên a không chia hết cho p

Do ñó n ⋮ p ⇒ n = mp

Thay vào (1) ta dễ dang suy ra ñược n = pl

Ví dụ 3. Cho p là số nguyên tố và a, n là các số nguyên dương. Chứng minh rằng nếu

2p + 3p = an

thì n = 1.

Lời giải

Nếu p = 2 thì 2p + 3p = 13 ⇒ n = 1

Nếu p > 2 ⇒ p lẻ

Ta có

2p + 3p ≡ 2p + (-2)p (mod 5) ≡ 0 (mod 5)

Do ñó a ⋮ 5

Giả sử n > 1.

Page 80: Lời nói ñầu - lovetoan.files.wordpress.com · Chứng minh rằng nếu a3 + b3 + c3 chia hết cho 9 thì một trong ba số a, b, c phải chia hết cho 3. Lời giải

Nguyễn Văn Thảo Chuyên ñề Số Học - Phần I

80

Khi ñó an ⋮ 25 và 32

32

++ pp

⋮ 5 ⇒ 2p - 1 - 2p - 2.3 + ... + 3p - 1 ⋮ 5

2p - 1 - 2p - 2.3 + ... + 3p - 1 ≡ p.2p - 1 (mod 5)

suy ra p.2p - 1 ⋮ 5 ⇒ p = 5 (do p là số nguyên tố)

Mà 25 + 35 = 753 ≠ an, ( với n > 1)

Vậy ñiều giả sử là sai.

Từ ñó ta có ñiều phải chứng minh.

III.1.4. Bài tập

Bài 1. Tìm các hàm số f: N → N thoả mãn ñòng thời các ñiều kiện sau

i) f(2n) = f(n) + n

ii) f(n) là số chính phương thì n là số chính phương

iii) f là hàm tăng nghiêm ngặt.

Bài 2. Tìm tất cảc các số nguyên dương n sao cho 2n + 1 và 3n + 1 ñều là số chính phương. Khi ñó chứng minh rằng n chia hết cho 40.

Bài 3. Chứng minh rằng tổng của 3, 4, 5, hoặc 6 số nguyên liên tiếp không thể là số chính phương.

Bài 4. (IMO 86) Cho d là một số nguyên khác 2, 5, 13. Chứng minh rằng luôn tìm ñược hai số nguyên a, b phân biệt trong tập {2, 5, 13, d} sao cho ab - 1 không phải số chính phương.

Bài 5. Tìm tất cả các số nguyên dương n sao cho luỹ thừa 4 của số ước của n chính bằng n.

Bài 6. Chứng minh rằng bình phương của một số nguyên lẻ luôn có dạng 8k + 1.

Bài 7. (Hungari 1998) Cho x, y, z là các số nguyên và z > 1. Chứng minh rằng

(x + 1)2 + (x + 2)2 + ... (x + 99)2 ≠ yz.

Bài 8. Chứng minh rằng với mọi số tự nhiên n thì giữa n2 và (n + 1)2 luôn tồn tại ba số tự nhiên phân biệt a, b, c sao cho a2 + b2 ⋮ c2.

Bài 9. Cho n là số tự nhiên có số ước số tự nhiên là s. Chứng minh rằng tích tất cả các

ước số ñó bằng n sn .

Bài 10. Tìm tất cả các cặp số nguyên dương x, y sao cho

(x + 1)y - 1 = x!

Bài 11. Tìm các số nguyên tố p sao cho tồn tại các số nguyên dương n, x, y thỏa mãn

x3 + y3 = pn.

Page 81: Lời nói ñầu - lovetoan.files.wordpress.com · Chứng minh rằng nếu a3 + b3 + c3 chia hết cho 9 thì một trong ba số a, b, c phải chia hết cho 3. Lời giải

Nguyễn Văn Thảo Chuyên ñề Số Học - Phần I

81

Bài 12. Cho n là số nguyên dương. Chứng minh rằng nếu 2n+1 và 3n + 1 là các số chính phương thì 5n + 3 không phải số nguyên tố.

Bài 13. Cho a và b là hai số nguyên dương. Số (36a + b)(a + 36b) có thể là luỹ thừa của 2 ñược hay không?

Bài 14. Cho tập A gồm các số nguyên dương và |A| > 3. Biết rằng tích ba phần tử bất kì của A ñều là số chính phương. Chứng minh rằng mọi phần tử của A ñều là số chính phương.

Bài 15. Cho n là một số nguyên dương sao cho 1282 2 +n là một số nguyên. Chứng

minh rằng 2 + 1282 2 +n là một số chính phương.

Bài 16. Cho a, b, c là các số nguyên dương sao cho

0 < a2 + b2 - abc ≤ c

Chứng minh rằng a2 + b2 - abc là một số chính phương.

Bài 17. Cho a và b là các số nguyên dương sao cho a2 + b2 chia hết cho ab + 1. Chứng minh rằng

1

22

++

ab

ba

là một số chính phương.

Bài 18. Cho x, y, z là các số nguyên dương.

Chứng minh rằng (xy + 1)(yz +1)(zx + 1) là các số nguyên dương khi và chỉ khi xy + 1, yz +1, zx + 1 ñều là số chính phương.

Bài 19. Cho a và b là các số nguyên sao cho với mọi số không âm n thì 2na = b ñều là số chính phương. Chứng minh rằng a = 0.

Bài 20. Cho p là một số nguyên dương lẻ. Chứng minh rằng tổng các luỹ thừa bậc p của p số nguyên liên tiếp chia hết cho p2.

Bài 21. Tìm các số nguyên dương n sao cho n.2n + 3n chia hết cho 5.

Bài 22. Tìm các số nguyên dương n sao cho n.2n + 3n chia hết cho 25.

Bài 23. Tìm các số tự nhiên n sao cho nn + 1 + (n + 1)n chia hết cho 5.

Bài 24. Tìm các số nguyên dương m, n, k lớn hơn 1 sao cho

1! + 2! +... + n! = mk.

Bài 25. Tìm các số nguyên tố có dạng 1722002 +n

(n là số tự nhiên) biểu diễn dưới dạng hiệu lập phương của hai số tự nhiên.

Page 82: Lời nói ñầu - lovetoan.files.wordpress.com · Chứng minh rằng nếu a3 + b3 + c3 chia hết cho 9 thì một trong ba số a, b, c phải chia hết cho 3. Lời giải

Nguyễn Văn Thảo Chuyên ñề Số Học - Phần I

82

III. 2. Áp dụng tổ hợp vào các bài toán số học

III.2.1. Một số ñịnh lí cơ bản

III.2.1.1. Nhị thức Newton

(a + b)n = ∑=

−n

k

kknkn baC

0

với a, b là các số thực tuỳ ý, n nguyên dương.)!(!

!

knk

nC k

n −=

III.2.1.2. ðịnh lí

Cho p là một số nguyên tố. Khi ñó

kpC ⋮ p với mọi p = 1, 2, ..., p - 1.

Chứng minh

Ta có

!

)1)...(1(

)!(!

!

k

kppp

kpk

pC p

n

+−−=

−=

Do p nguyên tố và k ∈ {1, 2, ..., p -1} nên (p, k!) = 1

Mà Cpk nguyên nên (p-1)(p-2) ...(p - k + 1) ⋮ k!

Hay

Zak

kppp∈=

+−−−!

)1)...(2)(1(

Từ ñó suy ra ñiều phải chứng minh.

III.2.1.3 Một số hệ thức cơ bản

1) knn

kn CC −=

2) 111−−− += k

nkn

kn CCC (Hệ thức Pascal)

3)

+

=<<< 21

2

1

10 ...n

n

n

nnn CCCC

4) nnnnn CCC 2...10 =+++

5) knm

ikm

k

i

in CCC +

=

=∑0

(Hệ thức Vandermon)

III.2.2. Các ví dụ và bài tập

Ví dụ 1. Chứng minh rằng với n là số nguyên dương lẻ thì tập

Page 83: Lời nói ñầu - lovetoan.files.wordpress.com · Chứng minh rằng nếu a3 + b3 + c3 chia hết cho 9 thì một trong ba số a, b, c phải chia hết cho 3. Lời giải

Nguyễn Văn Thảo Chuyên ñề Số Học - Phần I

83

S = }C,...,C,{C 2

1-n

n2n

1n

chứa lẻ các số lẻ.

Lời giải

ðặt

Sn = 2

121 ...

+++n

nnn CCC

Khi ñó

2Sn = 2...10 −+++ nnnn CCC = 2n - 2.

⇒ Sn = 2n - 1 - 1 là số lẻ

Vậy tập S phải chứa lẻ các số lẻ.

Ví dụ 2. (Trung Quốc - 1998) Tìm số tự nhiên n ≥ 3 sao cho

22000 ⋮ 1 + .321nnn CCC ++

Lời giải

Theo bài ta có

1 + knnn CCC 2321 =++ (0 ≤ k ≤ 2000, k nguyên)

⇔ knnn2

6

)6)(1( 2

=+−+

⇔ (n + 1)(n2 - n + 6) = 3.2k + 1

ðặt m = n + 1 (m ≥ 4)

Khi ñó ta có

m(m2 - 3m + 8) = 3.2k + 1.

Do ñó, chỉ có thể xảy ra một trong hai trường hợp sau:

+) Trường hợp 1: m = 2s

Do m ≥ 4 nên s ≥ 2

⇒ m2 - 3m + 8 = 22s - 3.2s + 8 = 3.2k + 1 - s.

Nếu s ≥ 4 thì 22s - 3.2s + 8 ≡ 8 (mod 16).

⇒ 8 ≡ 3.2k + l - s (mod 16) ⇒ 2k + 1 - s = 8 ⇒ m2 - 3m + 8 = 24

(không có nghiệm nguyên)

Nếu s = 2 ⇒ m = 4 ⇒ n = 3 (thỏa mãn)

Nếu s = 3 ⇒ m = 8 ⇒ n = 7 (thỏa mãn)

+) Trường hợp 2. m = 3.2s

Page 84: Lời nói ñầu - lovetoan.files.wordpress.com · Chứng minh rằng nếu a3 + b3 + c3 chia hết cho 9 thì một trong ba số a, b, c phải chia hết cho 3. Lời giải

Nguyễn Văn Thảo Chuyên ñề Số Học - Phần I

84

Làm tương tự như trên, ta tìm ñược n = 23.

Vậy n = 3, n = 7, n = 23 là những giá trị cần tìm.

Ví dụ 3. Cho p là một số nguyên tố. Chứng minh rằng

)(mod2 22 pC p

p ≡

Lời giải

Theo hệ thức Vandermon, ta có 0110

2 ... ppp

ppp

ppp

pp CCCCCCC +++= −

Mà kpC ⋮ p với mọi p = 1, 2, ..., p - 1.

Do ñó ip

pipCC − ⋮ p2 với mọi i = 1, 2, ..., p - 1

Từ ñó suy ra ñiều phải chứng minh.

Ví dụ 3. (Hungari - 2001) Cho m, n là các số nguyên dương và 1 ≤ m ≤ n. Chứng minh rằng

∑−

−1

1

)1(m

kn

k Cn ⋮ m.

Lời giải

Ta có

)()1(...)()(

)1...(1

1212

11

11

10

10

1

11210

−−

−−−−−−−

−−

+−+−+++−=

−−+−mn

mn

mnnnnn

mn

mnnn

CCCCCCC

CCCC

= (-1)m - 11

1−−

mnC

Do ñó

∑−

−1

1

)1(m

kn

k Cn = (-1)m - 1n 11−−

mnC = (-1)m-1.m m

nC ⋮ m.

Từ ñó có ñiều phải chứng minh.

Ví dụ 4. (VMO - 2011) Cho dãy số nguyên (an) xác ñịnh bởi

a0 = 1, a1 = - 1

an= 6an - 1 + 5an - 2 , với n = 2, 3, ...

Chứng minh rằng a2012 - 2010 chia hết cho 2011.

Lời giải

Dễ dàng tìm ñược số hạng tổng quát của dãy số là

Page 85: Lời nói ñầu - lovetoan.files.wordpress.com · Chứng minh rằng nếu a3 + b3 + c3 chia hết cho 9 thì một trong ba số a, b, c phải chia hết cho 3. Lời giải

Nguyễn Văn Thảo Chuyên ñề Số Học - Phần I

85

14

)143)(1427()143)(1427( nn

na−+++−

=

Do ñó

14

)143)(1427()143)(1427( 20122012

2012

−+++−=a

Mặt khác, theo khai triển Newton ta có

14)14.(3)143(2012

0

20122012

2012 BACk

k

kk +==+ ∑=

14)14.(3)1()143(2012

0

20122012

2012 BACk

k

kkk −=−=− ∑=

− .

Trong ñó

.14...14.33 100620122012

201022012

201202012 CCCA +++=

.14.3....14.33 100520112012

200932012

201112012 CCCB +++=

Do ñó

14

)14)(1427()14)(1427(2012

BABAa

−+++−=

⇔ a2012 = A + 4B.

Bây giờ ta chỉ cần chứng minh A + 4B ≡ -1 (mod 2011)

Thật vậy, ta có 1

201120112012−+= kkk CCC với mọi k = 2, 3, ..., 2010

Mà 2011 là số nguyên tố nên kC2011 ⋮ 2011 với mọi k = 1, 2, ..., 2010.

Từ ñó suy ra kC2012 ⋮ 2011 với mọi k = 2, 3, ..., 2010.

Do ñó A + 4B ≡ 32012 + 141006 - 4(32011 + 3.141005) (mod 2011)

≡ 2. 141005 - 32011 (mod 2011)

Mà 32011 ≡ 3 (mod 2011)

141005 ≡ 20251005 ≡ (452)1005 ≡ 452010 ≡ 1 (mod 2011).

Từ ñó suy ra A + 4B ≡ 2 - 3 (mod 2011) ≡ -1 (mod 2011)

Vậy ta có ñiều phải chứng minh.

Page 86: Lời nói ñầu - lovetoan.files.wordpress.com · Chứng minh rằng nếu a3 + b3 + c3 chia hết cho 9 thì một trong ba số a, b, c phải chia hết cho 3. Lời giải

Nguyễn Văn Thảo Chuyên ñề Số Học - Phần I

86

III.2.3 Bài tập

Bài 1. Tìm ước chung lớn nhất của 20052006

32006

12006 ,...,, CCC .

Bài 2. (T7/245) Cho các số tự nhiên m, n, p thỏa mãn ñiều kiện p ≤ m + n.

Chứng minh rằng ∑=

−−+b

ai

ipm

inCCpbnm !)!( chia hết cho (m + n - a)!

Trong ñó a = max {0, p - m}, b = min {p, n}.

Bài 3. Cho xn = [ ]n)154( + , với [x] là phần nguyên của x, n là số tự nhiên.

Tìm số dư của xn khi chia cho 8.

Bài 4. Chứng minh rằng số

∑=

−=1004

0

2200822009 32

k

kkkCS

là tổng của hai số chính phương liên tiếp.

Bài 5. Chứng minh rằng với mọi số nguyên dương n ta có

)1(2 +nC nn ⋮

Bài 6. Tìm cặp số tự nhiên n, k sao cho kn

n nC )3(3 = .

Bài 7. Cho m và n là các số nguyên dương, m lẻ. Chứng minh rằng

∑=

−m

k

kkmm

nCn 0

33 )13(

3.

1

là một số nguyên.

Bài 8. Chứng minh rằng với mọi số nguyên dương n thì số

∑=

++

n

k

kknC

0

31212 2

không chia hết cho 5.

Page 87: Lời nói ñầu - lovetoan.files.wordpress.com · Chứng minh rằng nếu a3 + b3 + c3 chia hết cho 9 thì một trong ba số a, b, c phải chia hết cho 3. Lời giải

Nguyễn Văn Thảo Chuyên ñề Số Học - Phần I

87

III. 3 Tính chất số học của dãy số nguyên Trong phần này tôi xin giới thiệu cùng bạn ñọc một số tính chất số học cơ bản của dãy số nguyên như tính chia hết, tính chính phương, tìm số hạng nguyên tố, ... Tuy nhiên, phần sâu hơn sẽ ñược tác giả giới thiệu trong chuyên ñề tiếp theo, sẽ viết riêng về dãy số nguyên.

III.3.1. Tính chia hết trong dãy số nguyên.

Ví dụ 1. Cho dãy số {un} thỏa mãn

=++=

==

−+ ,...3,2,2254

50,7

11

21

nuuu

uu

nnn

Chứng minh rằng u1996 chia hết cho 1997.

Lời giải

Xét dãy {vn} thỏa mãn vn = 4un + 11 khi ñó

=+=

==

−+ ,...3,2,54

211,39

11

21

nvvv

vv

nnn

Dựa vào phương trình ñặc trưng của dãy {vn} ta dễ dàng xác ñịnh ñược số hạng tổng quát của dãy là

3

5.25)1(8 nn

nv+−

=

Do ñó

3

5.258 1996

1996

+=v

Vì 1997 là số nguyên tố nên theo ñịnh li Fermat, ta có

51996 ≡ 1 (mod 1997)

⇒ 8 + 25.51996 ≡ 8 + 25 (mod 1997) ≡ 33 (mod 1997).

⇒ 3v1996 ≡ 33 (mod 1997).

Mà v1996 = 4u1996 + 11 nên suy ra

12u1996 + 33 ≡ 33 (mod 1997)

⇒ 12u1996 ⋮ 1997

Mặt khác (12, 1997) = 1 nên u1996 ⋮ 1997.

Vậy ta có ñiều phải chứng minh.

Ví dụ 2. Cho dãy số {un} xác ñịnh như sau

=−=

−===

−−+ ,...4,367

18,14,0

211

321

nuuu

uuu

nnn

Page 88: Lời nói ñầu - lovetoan.files.wordpress.com · Chứng minh rằng nếu a3 + b3 + c3 chia hết cho 9 thì một trong ba số a, b, c phải chia hết cho 3. Lời giải

Nguyễn Văn Thảo Chuyên ñề Số Học - Phần I

88

Chứng minh rằng với mọi số nguyên tố p thì up ⋮ p.

Lời giải

Theo cách xác ñịnh dãy số ta tìm ñược số hạng tổng quát của un là

un= 1 + 2n + (-3)n.

Với mọi số nguyên tố p thì

2p ≡ 2 (mod p) và (-3)p ≡ -3 (mod p)

Do ñó

up = 1 + 2p + (-3)p ≡ 1 + 2 + (-3) (mod p)

Hay up ⋮ p.

Vậy ta có ñiều phải chứng minh.

Nhận xét: Với những dãy số nguyên tuyến tính có phương trình ñặc trưng có nghiệm nguyên thì ta dễ dàng sử dụng ñịnh lí Fermat trong chứng minh chia hết. Còn với những dãy nguyên tuyến tính có phương trình ñặc trưng có nghiệm vô tỉ thì bao giờ ta cũng phải sử dụng nhị thức Newton ñể chứng minh chia hết.

Ví dụ 3. (VMO - 2011) Cho dãy số nguyên (an) xác ñịnh bởi

a0 = 1, a1 = - 1

an= 6an - 1 + 5an - 2 , với n = 2, 3, ...

Chứng minh rằng a2012 - 2010 chia hết cho 2011.

Lời giải

Dễ dàng tìm ñược số hạng tổng quát của dãy số là

14

)143)(1427()143)(1427( nn

na−+++−

=

Do ñó

14

)143)(1427()143)(1427( 20122012

2012

−+++−=a

Mặt khác, theo khai triển Newton ta có

14)14.(3)143(2012

0

20122012

2012 BACk

k

kk +==+ ∑=

14)14.(3)1()143(2012

0

20122012

2012 BACk

k

kkk −=−=− ∑=

− .

Trong ñó

.14...14.33 100620122012

201022012

201202012 CCCA +++=

.14.3....14.33 100520112012

200932012

201112012 CCCB +++=

Page 89: Lời nói ñầu - lovetoan.files.wordpress.com · Chứng minh rằng nếu a3 + b3 + c3 chia hết cho 9 thì một trong ba số a, b, c phải chia hết cho 3. Lời giải

Nguyễn Văn Thảo Chuyên ñề Số Học - Phần I

89

Do ñó

14

)14)(1427()14)(1427(2012

BABAa

−+++−=

⇔ a2012 = A + 4B.

Bây giờ ta chỉ cần chứng minh A + 4B ≡ -1 (mod 2011)

Thật vậy, ta có 1

201120112012−+= kkk CCC với mọi k = 2, 3, ..., 2010

Mà 2011 là số nguyên tố nên kC2011 ⋮ 2011 với mọi k = 1, 2, ..., 2010.

Từ ñó suy ra kC2012 ⋮ 2011 với mọi k = 2, 3, ..., 2010.

Do ñó A + 4B ≡ 32012 + 141006 - 4(32011 + 3.141005) (mod 2011)

≡ 2. 141005 - 32011 (mod 2011)

Mà 32011 ≡ 3 (mod 2011)

14 là số chính phương mod 2011 nên 141005 = 2

12011

14−

≡ 1 (mod 2011).

Từ ñó suy ra A + 4B ≡ 2 - 3 (mod 2011) ≡ -1 (mod 2011)

Vậy ta có ñiều phải chứng minh.

Ví dụ 4. Cho dãy số {un} ñược xác ñịnh như sau

=+=

==

−+ ,...3,2,2

6,2

11

21

nuuu

uu

nnn

Tìm phần dư của u1024 chia cho 1023.

Lời giải

Dễ dàng tìm ñược số hạng tổng quát của dãy {un} là nn

nu )21()21( −++=

Do ñó 10241024

1024 )21()21( −++=u

Theo khai triển nhị thức Newton, ta có

2)21(,2)21( 10241024 BABA −=−+=+

Với 10241024

51241024

221024

01024 2...22 CCCCA ++++=

10231024

51251024

231024

11024 2...22 CCCCB ++++=

Page 90: Lời nói ñầu - lovetoan.files.wordpress.com · Chứng minh rằng nếu a3 + b3 + c3 chia hết cho 9 thì một trong ba số a, b, c phải chia hết cho 3. Lời giải

Nguyễn Văn Thảo Chuyên ñề Số Học - Phần I

90

Do ñ ó u1024 = 2A

Ta có 1

102310231024−+= kkk CCC với mọi k = 2, 3, ..., 1022.

Mà 1023 là số nguyên tố nên kC1023 ⋮1023 với mọi k = 1, 2, ..., 1022.

Từ ñó suy ra kC1024 ⋮ 1023 với mọi k = 2, 3, ..., 1022.

Nên

)1023(mod222 10241024

51301024 CCA +≡

≡ 2+2513 (mod 1023).

Mặt khác

210 ≡ 1 (mod 1023) ⇒ 2513 = 8.(210)51 ≡ 8 (mod 1023)

Do ñó 2A ≡ 10 (mod 1023)

Vậy phần dư của phép chia u1024 cho 1023 là 10.

Nhận xét: Việc sử dụng nhị thức Newton rất có hiệu quả ñối với các bài toán chứng minh một số hạng cụ thể của dãy chia hết cho p. Tuy nhiên nó thường không có tác dụng ñối với việc chứng minh trong dãy số tồn tại một số hạng của dãy chia hết cho p hoặc tồn tại vô hạn số hạng của dãy chia hết cho p. ðối với những bài toán dạng này ta thường phải chứng minh dãy số dư của un cho p tuần hoàn kể từ một số hạng nào ñó.

Ví dụ 5. Cho dãy số {un} ñược xácñịnh như sau

=−+=

==

−+ ,...3,2,197654

100,20

11

21

nuuu

uu

nnn

Chứng minh rằng tồn tại ít nhất một số hạng của dãy chia hết cho 1996.

Lời giải

Xét dãy {vn} với vn là phần dư của phép chia un cho 1996.

Hay vn ≡ un (mod 1996), 0 ≤ vn ≤ 1995.

Dễ thấy vn+1 ≡ 4vn + 5vn - 1 - 1976 (mod 1996)

Xét các cặp (v1, v2), (v2, v3), ..., (vn, vn + 1), ...

Vì dãy {vn} là vô hạn mà số các cặp (a, b) với 0 ≤ a, b, ≤ 1995 là hữu hạn nên phải tồn tại hai số tự nhiên i, j (i > j) sao cho

=

=

++ 11 ji

ji

vv

vv

Khi ñó. Ta có

Page 91: Lời nói ñầu - lovetoan.files.wordpress.com · Chứng minh rằng nếu a3 + b3 + c3 chia hết cho 9 thì một trong ba số a, b, c phải chia hết cho 3. Lời giải

Nguyễn Văn Thảo Chuyên ñề Số Học - Phần I

91

5(vi - 1 - vj - 1 ) ≡ 5(vi +1 - 4vi + 1976 ) - 5(vj+1 - 4vj + 1996)

≡ 5(vi+1 - vj+1) - 20(vi - vj) (mod 1996)

≡ 0 (mod 1996)

Mà (5, 1996) = 1 nên vi - 1 - vj - 1 ≡ 0 (mod 1996)

Vì 0 ≤ vi - 1 , vj - 1 ≤ 1995 nên vi - 1 = vj - 1

Lí luận tương tự ta dẫn ñến vi - 2 = vj - 2

Cứ tiếp tục như vậy ta sẽ dẫn ñến

=

=

−+

−+

)(11

)(22

ji

ji

vv

vv

Bây giờ ta sẽ chứng minh vi - j = 0

Thật vậy, ta có

5vi - j ≡ vi - j + 2 - 4vi - j + 1 + 1976 (mod 1996)

≡ v2 - 4v1 + 1976 (mod 1996)

≡ u2 - 4u1 + 1976 (mod 1996)

≡ 100 - 4.20 + 1976 (mod 1996)

≡ 0 (mod 1996).

Vì (5, 1996) = 1 nên vi - j ≡ 0 (mod 1996)

Do ñó ui - j ≡ 0 (mod 1996)

Vậy có ñiều phải chứng minh.

Ví dụ 6. (Chọn ðT Khoa học tự nhiên - 2009)

Cho dãy {an} ñược xác ñịnh

a0 = 0, a1 = 1, a2 = 2, a3 = 6

an + 4 = 2an + 3 + an + 2 - 2an + 1 - an, n = 0, 1, ...

a) Chứng minh rằng an chia hết cho n với mọi n ≥ 1.

b) Chứng minh rằng dãy số

n

an , n = 1, 2, ... chứa vô số số hạng chia hết cho

2009.

Lời giải

a) Phương trình ñặc trưng của dãy {an} là

x4 - 2x3 - x2 + 2x +1 = 0

⇔ (x2 - x - 1)2 = 0

Page 92: Lời nói ñầu - lovetoan.files.wordpress.com · Chứng minh rằng nếu a3 + b3 + c3 chia hết cho 9 thì một trong ba số a, b, c phải chia hết cho 3. Lời giải

Nguyễn Văn Thảo Chuyên ñề Số Học - Phần I

92

=−

=

=+

=

β

α

2

51

2

51

x

x

Khi ñó số hạng tổng quát của dãy trên là

an = c1αn + c2β

n + n(c3αn + c4β

n)

Dễ dàng tìm ñược

c1 = c2 = 0, 5

1,

5

143 −== cc .

Do ñó

)5

1

5

1( nn

n na βα −=

Từ ñó suy ra nn F

n

a= với {Fn} là dãy Fibonaci.

Do ñó an ⋮ n với mọi n.

b) Bổ sung vào dãy {Fn} một số hạng F0 = 0 mà ta vẫn giữ ñược hệ thức

Fn +1 = Fn + Fn - 1 , n = 0, 1, ...

Xét dãy {vn} với vn là phần dư của phép chia Fn cho 2009.

Hay vn ≡ un (mod 2009), 0 ≤ vn ≤ 2008.

Dễ thấy vn+1 ≡ vn + vn - 1 (mod 2009)

Xét các cặp (v1, v2), (v2, v3), ..., (vn, vn + 1), ...

Vì dãy {vn} là vô hạn mà số các cặp (a, b) với 0 ≤ a, b ≤ 2008 là hữu hạn nên phải tồn tại hai số tự nhiên i, j (i > j) sao cho

=

=

++ 11 ji

ji

vv

vv

Làm tương tự ví dụ 5, ta suy ra 0 = v0 = vi - j

và dãy {vn} tuần hoàn với chu kì (i - j).

Do ñó vk(i - j) = 0 hay Fk(i - j) ≡ 0 (mod 2009)

Vậy ta có ñiều phải chứng minh.

Page 93: Lời nói ñầu - lovetoan.files.wordpress.com · Chứng minh rằng nếu a3 + b3 + c3 chia hết cho 9 thì một trong ba số a, b, c phải chia hết cho 3. Lời giải

Nguyễn Văn Thảo Chuyên ñề Số Học - Phần I

93

III.3. 2. Dãy số và số chính phương

Ví dụ 1. Cho dãy số {an} ñược xác ñịnh như sau

=−=+−

==

−+ ,...2,1,)1((2

3

1,0

11

10

naaa

aa

nnnn

Chứng minh rằng với mọi n thì an là số chính phương.

Lời giải

Ta sẽ chứng minh với mọi n thì an = Fn2, trong ñó {Fn} là dãy Fibonaci

Thật vậy

a0 = 1 = F02, a1 = 1 = F1

2, a2 = 4 = F22.

Giả sử ak = Fk2 với mọi k ≤ n

Khi ñó

an = Fn

2, an - 1 = (Fn-1)

2, an - 2 = (Fn - 2)2

Theo cách xác ñịnh dãy {an} ta có

an + 1-3an + an - 1 = 2(-1)n

an - 3an - 1 + an - 2 = 2(-1)n - 1

Từ ñó ta suy ra

an+1 - 2an - 2an - 1 + an - 2 = 0

⇔ an +1 = 2(Fn)2 + 2(Fn - 1)

2 - (Fn - 2)2

= (Fn + Fn - 1)2 + (Fn - Fn - 1)

2 - (Fn - 2)2

= (Fn + 1)2 +(Fn - 2)

2 - (Fn - 2)2 = (Fn + 1)

2

Từ ñó suy ra diều phải chứng minh.

Ví dụ 2. Cho dãy số {an} xác ñịnh như sau

=−=

==

−+ ,...2,1,3

3,2

11

10

naaa

aa

nnn

Tìm n sao cho 5(an - 2) là một số chính phương.

Lời giải

Dễ dàng xác ñịnh ñược số hạng tổng quát của dãy trên là

nnna )

2

53()

2

53(

−+

+=

Mặt khác ta cũng chứng minh ñược ngay

Page 94: Lời nói ñầu - lovetoan.files.wordpress.com · Chứng minh rằng nếu a3 + b3 + c3 chia hết cho 9 thì một trong ba số a, b, c phải chia hết cho 3. Lời giải

Nguyễn Văn Thảo Chuyên ñề Số Học - Phần I

94

a2k ≡ 2 (mod 5) và a2k + 1 ≡ 3 (mod 5).

Do ñó 5(an - 2) là số chính phương thì an - 2 ⋮ 5 ⇒ n = 2k.

Khi ñó

kkka 22

2 )2

53()

2

53(

−+

+=

Khi ñó

5(a2k - 2) = 2

)2

53()

2

53(5

−−

+ kk

Theo khai triển Newton ta có

k

kk

k BABA

2

5)

2

53(,

2

5)

2

53(

−=

−+=

+

Với

−−

−−

+++=

+++=

21

221

22

3311

222

22

220

53...5.33

5.3...5.33kk

kk

kk

kk

k

kkk

k

kk

kk

k

CCCB

CCCA

Khi ñó kkkk c

B.5

2

5)

2

53()

2

53(

1==

−−

+−

với ck là số hữu tỉ.

⇒ a2k - 2 = 5ck2 nguyên nên ck

2 là số nguyên

Mà ck hữu tỉ nên ck ∈ Z

Từ ñó suy ra 5(a2k - 2) = (5ck)2 với 5ck ∈ Z

Vậy với mọi k thì 5(a2k - 2) là số chính phương.

Ví dụ 3. Cho dãy số {an} ñược xác ñịnh như sau

an = 28 + 211 + 2n, n = 1, 2, ...

Tìm n ñể an là một số chính phương.

Lời giải

Ta có

an = 28 + 211 + 2n = x2

⇔ 2n = (x - 48)(x + 48)

Với x là một số nguyên dương nào ñó.

Khi ñó

x - 48 = 2s và x + 48 = 2n - s, n > 2s

Từ ñó suy ra

Page 95: Lời nói ñầu - lovetoan.files.wordpress.com · Chứng minh rằng nếu a3 + b3 + c3 chia hết cho 9 thì một trong ba số a, b, c phải chia hết cho 3. Lời giải

Nguyễn Văn Thảo Chuyên ñề Số Học - Phần I

95

2n - s - 2s = 96.

⇔ 2s(2n - 2s - 1) = 3.25.

Từ ñó suy ra

=

=⇔

=−

=− 12

5

312

52 n

sssn

Vậy an là số chính phương khi và chỉ khi n = 12

Ví dụ 4. Cho dãy số {an} ñược xác ñịnh như sau

=+−+=

==

−+ ,...3,2)1()2(

3,1

11

21

nanana

aa

nnn

Tìm n ñể an là luỹ thừa p của một số nguyên (p ≥ 2).

Lời giải

Ta dễ dàng chứng minh ñược

an = 1! + 2! + ... + n!

Trường hợp 1: p = 2.

Ta có n! ⋮ 10 với mọi n ≥ 5

Do ñó, nếu n ≥ 5 thì an ≡ 1! + 2! + 3! + 4! (mod 10)

≡ (1 + 2 + 6 + 24) (mod 10) ≡ 3 (mod 10)

Từ ñó suy ra an không thể là số chính phương nếu n ≥ 5.

Với 1 ≤ n ≤ 4, thử trực tiếp thấy n = 1 và n = 3 thỏa mãn.

Trường hợp 2. n ≥ 3

Ta có a1 = 1 = 1p với mọi p.

Với n ≥ 2 thì an = 1! + 2! + 3! +...+ n! ⋮ 3

Giả sử an = bp (p ≥ 3) ⇒ un ⋮ 27

Ta có 9! ⋮ 27 nên k! ⋮ 27 với mọi k ≥ 9.

Do ñó

an ≡ a8 (mod 27) với n ≥ 9.

a8 = 1! + 2! + ... +8! = 46233 ≡ 1(mod 27).

Nên an không chia hết cho 27 với mọi n ≥ 8 hay an ≠ bp với mọi n ≥ 8.

Bằng cách thử trực tiếp k = 2, 3, ..., 7 thấy không có giá trị nào thỏa mãn.

Vậy n = 1 là giá trị cần tìm.

Page 96: Lời nói ñầu - lovetoan.files.wordpress.com · Chứng minh rằng nếu a3 + b3 + c3 chia hết cho 9 thì một trong ba số a, b, c phải chia hết cho 3. Lời giải

Nguyễn Văn Thảo Chuyên ñề Số Học - Phần I

96

III.3.3. Dãy số nguyên số nguyên tố.

Ví dụ 1. Cho dãy số {un} ñược xác ñịnh như sau

un = 3n - 2n - 1

Chứng minh rằng nếu p là số nguyên tố thì up ⋮ 42p.

Lời giải

Ta có

up = 3p – 2p – 1 = (3p - 3) – (2p - 2) ≡ 0 (mod p). (1)

Mặt khác

3p – 2p – 1 = (3p - 1) – 2p ⋮ 2 (2)

Mà p > 7 ⇒ p lẻ

Do ñó

3p – 2p – 1 ≡ - (-1)p – 1 ≡ 0 (mod 3) (3)

Bây giờ ta cần chứng minh 3p – 2p – 1 ⋮ 7

Ta có

3p – 2p - 1 = 3.3p-1 – 2p – 1

=1 1

2 23.9 2 1 3.2 2 1p p

p p− −

− − ≡ − − (mod 7)

= 1 1

2 22 2 2 1p p

p+ −

+ − −

Do (p, 3) = 1 nên p = 3k + 1 hoặc p = 3k + 2

Nếu p = 3k + 1 thì 1 1

2 22 2 2 1p p

p+ −

+ − − = 1 1

2 28 1 2 2 2 2 (mod 7)p p

k p p+ +

− + − ≡ −

1 1 1

2 2 22 (2 1) 2 (8 1) 0(mod 7)p p p

k+ − +

= − = − ≡

Nếu p = 3k + 2 ta chứng minh tương tự.

Từ ñó suy ra ñpcm.

Ví dụ 2. [IMO - 2005] Cho dãy số (an) xác ñịnh như sau

an = 2n + 3n + 6n – 1 với n = 1, 2, …

Tìm số tự nhiên nguyên tố cùng nhau với mọi số hạng của dãy trên.

Lời giải

Ta sẽ chứng minh rằng với mọi số nguyên tố p ñều tồn tại một số hạng an chia hết cho p.

Thật vậy, ta có a2 = 22 + 32 + 62 – 1 = 48 chia hết cho 2 và 3.

Page 97: Lời nói ñầu - lovetoan.files.wordpress.com · Chứng minh rằng nếu a3 + b3 + c3 chia hết cho 9 thì một trong ba số a, b, c phải chia hết cho 3. Lời giải

Nguyễn Văn Thảo Chuyên ñề Số Học - Phần I

97

Xét p ≥ 5

Ta có

(2, p) = 1; (3, p) = 1; (6, p) = 1

Do ñó, từ ñịnh lí Fermat suy ra

2p – 1 ≡ 3p – 1 ≡ 6p – 1 ≡ 1 (mod p)

Từ ñó dễ dàng chứng minh ñược 6ap – 2 ⋮ p

Mà (p, 6) = 1 nên ap – 2 ⋮ p

Do ñó chỉ có số 1 là số tự nhiên duy nhất nguyên tố cùng nhau với mọi số hạng của dãy (an).

III.3.4 Bài tập

Bài 1. Cho x1, x2 là nghiệm của phương trình

x2 - 6x +1 = 0.

Chứng minh rằng với mọi n ∈ N thì x1n + x2

n là một số nguyên và không chia hết cho 5.

Bài 2. Cho dãy {un} thỏa mãn

,...5,4,86

24,2,1

32

2213

21

321

=

−=

===

−−

−−−−n

uu

uuuuu

uuu

nn

nnnnn

Chứng minh rằng với mọi n thì un chia hết cho n.

Bài 3. (T6/254) Cho dãy số {xn} xác ñịnh như sau:

≥−=

==

−+ .2,35

,

12

1

10

nxxx

bxax

nnn

với a, b là hai số nguyên cho trước. Chứng minh rằng dãy số trên hoặc không có số hạng nào chia hết cho 1997 hoặc có vô số số hạng chia hết cho 1997.

Bài 4. (T6/255) Cho dãy {an} ñược xác ñịnh như sau

≥−−=

==

+++ .1,19947530

96,1964

12

12

21

naaaaa

aa

nnnnn

Chứng minh rằng không tồn tại số hạng nào của dãy là tổng luỹ thừa bảy của ba số nguyên.

Bài 5. Cho dãy {un} ñược xác ñịnh bởi:

≥+=

==

++ .0,72

11,3

12

10

nuuu

uu

nnn

Page 98: Lời nói ñầu - lovetoan.files.wordpress.com · Chứng minh rằng nếu a3 + b3 + c3 chia hết cho 9 thì một trong ba số a, b, c phải chia hết cho 3. Lời giải

Nguyễn Văn Thảo Chuyên ñề Số Học - Phần I

98

Tìm các số nguyên dương lẻ a sao cho với các số nguyên dương m và n tuỳ ý luôn tìm ñược số nguyên dương k sao cho au k

n − chia hết cho 2m.

Bài 6. Cho dãy {un} ñược xác ñịnh như sau:

≥−=

==

++ 0,1999

1

12

10

nuuu

uu

nnn

Tìm các số tự nhiên n sao cho un là số nguyên tố.

Bài 7. (Bulgari 1999) Cho {an} là dãy số nguyên thỏa mãn

(n - 1)an + 1 = (n + 1)an - 2(n - 1).

Biết rằng a1999 chia hết cho 2000. Tìm số nguyên dương n ≥ 2 và n nhỏ nhất sao cho an chia

hết cho 2000.

Bài 8. Cho một dãy số gồm 2n + 1 số nguyên dương liên tiếp sao cho tổng bình phương của n +1 hạng ñầu tiên bằng tổng bình phương của n số hạng còn lại. Hỏi trong dãy ñó có số 2001 không?

Bài 9. Cho dãy {an} xác ñịnh như sau

=+=

==

++ ,...1,0,2

1,0

12

10

naaa

aa

nnn

Chứng minh rằng an chia hết cho 2k khi và chỉ khi n chia hết 2k, với mọi k nguyên không âm.

Bài 10. Cho dãy số {un} thỏa mãn

≥+=

==

++ .172

11,3

12

21

nuuu

uu

nnn

Tìm số nguyên dương lẻ a sao cho với mọi m, n nguyên dương tuỳ ý luôn tìm ñược số

nguyên dương k sao cho au kn − chia hết cho 2m.

Bài 11. Cho dãy {un} ñược xác ñịn như sau

=−=

==

++ ...,1,0,1999

1,0

12

10

nuuu

uu

nnn

Tìm các số tự nhiên n sao cho un là số nguyên tố.

Page 99: Lời nói ñầu - lovetoan.files.wordpress.com · Chứng minh rằng nếu a3 + b3 + c3 chia hết cho 9 thì một trong ba số a, b, c phải chia hết cho 3. Lời giải

Nguyễn Văn Thảo Chuyên ñề Số Học - Phần I

99

TÀI LIỆU THAM KHẢO [1] ðặng Hùng Thắng, Nguyễn Văn Ngọc, Vũ Kim Thuỷ - NXBGD, 1997.

[2] Nguyễn Vũ Lương, Nguyễn Ngọc Thắng, Nguyễn Lưu Sơn, Phạm Văn Hùng - NXB ðại Học Quốc Gia Hà Nội, 2006.

[3] Phan Huy Khải - Các chuyên ñề Số học - NXBGD, 2005.

[4] Nguyễn Văn Mậu, Trần nam Dũng, ðặng Hùng Thắng, ðặng Huy Ruận - NXBGD, 2008.

[5] Nguyễn Sinh Nguyên, Nguyễn Văn Nho, Lê Hoành Phò - Tuyển tập các bài dự thi Olimpiad Toán học Quốc tế 1991 - 2001 - NXBGD, 2001.

[6] Titu Andreescu, Dorin Andrica, Zuming Feng - 104 Number theory problems from the training of the USA IMO team - NXB Birkhauser, 2006.

[5] Tạp chí Toán học và tuổi trẻ

[6] Các ñề thi vô ñịch các nước.

[7] Các tài liệu trên mạng Internet.